You are on page 1of 32

Case Based 3 2010

1.An 18-year-old accidentally drinks antifreeze. Within a half an hour, he starts to


experience nausea, vomiting, and headache. He becomes very stuporous. He then has
a seizure. His friends realize something is wrong, so they summon the paramedics.
What problem would be expected?
a) metabolic acidosis
b) respiratory alkalosis
c) Metabolic alkalosis
d) Respiratory acidosis

Rationale: Antifreeze is ethylene glycol. It can produce an elevated anion gap


metabolic acidosis. When a metabolic acidosis exists, evaluating the anion gap is
useful. Sodium and potassium are the positive ions within the extracellular fluid.
Chloride and bicarbonate are the negative ions in the extracellular fluid. Normally,
there are more measured positive ions than measured negative ions. The anion gap
is this difference. (Na+ + K+) - (Cl- + HCO3-) = anion gap Note: the potassium is
frequently ignored and left out of the equation. The accumulation of another anion,
as occurs with ethylene glycol toxicity, will result in an elevated anion gap metabolic
acidosis. A normal anion gap acidosis is seen where there is not the addition of an
excess organic acid, as in a hyperchloremic acidosis. In a hyperchloremic acidosis,
bicarbonate is decreased and chloride ion is increased. This is a normal anion gap
acidosis. Metabolic alkalosis is due to the addition of a base or the loss of acid.
Ethylene glycol toxicity does not result in a metabolic alkalosis. Respiratory acidosis
occurs with changes in ventilation and breathing. Alkalemia refers to changes in
blood pH towards the basic side (higher pH). References: 1. Andreoli, Tomas E.,
M.D., Carpenter, Charles, C.J., M.D., Griggs, Robert C., M.D., Benjamin, Ivor J.,
M.D. Andreoli and Carpenter's Cecil Essentials of Medicine. 7th edition. Philadelphia,
PA. 2007. 2. C. Maccari, K.S. Kamel, M.R. Davids and M.L. Halperin. The patient
with a severe degree of metabolic acidosis: a deductive analysis QJM: An
International Journal of Medicine. Volume 99, Number 7. Pp. 475-485

2.A 50-year-old man comes to you with an 8-year history of uncontrolled hypertension
and recent onset of chest pain during exertion. His blood pressure is 150/95 mmHg, and
based on investigations including a stress test and an echocardiogram, a diagnosis of
stable angina was made. All the blood parameters were normal. Even with beta blockers
at maximal dosage, the angina persisted. What is the most appropriate drug to add to
this patient’s regimen?
a) Angiotensin-Converting enzyme inhibitors (ACEI)
b) Aldosterone antagonists
c) Angiotensin Receptor Blockers (ARB)
d) Calcium channel blockers (CCB)
e) Central alpha 2 agonists

Rationale: CCBs prevent calcium entry into the cell, thereby preventing the involuntary
muscle cells of the blood vessels from contracting and in effect allowing the coronary
arteries and arterioles to relax and dilate, enabling more blood to the heart muscles.
Although beta blockers are recommended as initial therapy for stable angina in the
absence of contraindications, if they cause unacceptable side effects or if the angina
persists, calcium channel blockers should be used. Beta blockers and Angiotensin-
Converting enzyme inhibitors (ACEI) are used in acute coronary syndromes comprised
of unstable angina or myocardial infarction. Aldosterone antagonists along with beta
blockers and ACEI are used in post-myocardial infarction patients. Angiotensin Receptor
Blockers (ARBs) are known to reduce the progression of renal disease and hence may
be used in angina associated with renal complications. Central alpha 2 agonists such as
clonidine stimulate the alpha 2 adrenergic receptors in the brain. This results in
decreased cardiac output and peripheral resistance. These drugs may cause fluid
retention and rebound hypertension if they are withdrawn abruptly. References: 1.
Seventh Report of the Joint National Committee on Prevention, Detection, Evaluation
and Treatment of High Blood Pressure (JNC 7) Express. National Heart, Lung, and
Blood Institute. Bethesda, Md. 2003. JAMA 2003;289:2560-71 2. Fihn SD, Williams SV,
Daley J, et al. Guidelines for the management of patients with chronic stable angina:
treatment. Ann Intern Med. 2001 Oct 16;135(8 Pt 1):616-32

3.A 54-year-old man complains of severe low back pain and right-sided sciatica
persisting for 3 months. A radiograph of the lumbar spine revealed moderate disc space
narrowing at L5/S1. The MRI showed a right L5/S1 paracentric disc herniation causing
posterolateral displacement of the right S1 nerve root. Which of the following statements
concerning this condition is true?
a) Most patients will require surgery
b) It is more common in the cervical spine
c) Herniated discs are in a final stage of degeneration
d) Changes in bowel and/or bladder function may occur
e) Radiographs are the most common method of diagnosis

Rationale: Changes in bowel and/or bladder function may occur in herniated disc. A
ruptured or herniated disc can be caused by injury or strain. The disc material will
degenerate normally as people age. The discs are round and flat, and have a tough
outer layer called the annulus. The center is called the nucleus and has a jelly-like
consistency. A portion of the nucleus can push out of the annulus through a tear or
rupture. The intervertebral disc is an avascular structure. A herniated (also called a
slipped or ruptured) disc is a fragment of the disc nucleus which is pushed out of the
annulus, into the spinal canal through a tear or rupture. Discs that become herniated are
usually in an early stage of degeneration. The spinal canal has limited space which is
inadequate for the spinal nerve and the displaced herniated disc fragment. Due to this
displacement, the disc presses on spinal nerves, often producing pain, which may be
severe. Risk factors include excessive body weight, poor muscle tone in the back and
abdominal muscles, poor lifting technique, and smoking. Herniated discs can occur in
any part of the spine. Herniated discs are more common in the lumbar spine, but also
occur in the cervical spine. Sequestration is the final stage of degeneration of the disc. In
this stage, the portion of the material separated from the disc itself may be freely floating
in the spinal canal. A herniated disc in the lumbar spine can often result in sciatica. This
is caused by pressure on one or more of the nerves or nerve roots in the affected area.
Symptoms may include pain, burning, tingling, leg weakness and numbness that can
radiate down through the leg and into the foot. Patients may also complain of loss of
bowel or bladder function. Symptoms may be bilateral or unilateral. Pain is often
described as being sharp, and can become more severe when the patient stands, sits,
or walks. It is usually the changing from one position to another that triggers the most
pain.
4.A 63-year-old white male has started to have episodes of chest pain that awaken him
at night. The doctor tells this patient that he has variant angina. What finding on an EKG
done during an episode of his pain would be most consistent with variant angina?
a) ST segment elevation
b) ST segment depression
c) Q waves
d) Tachycardia
e) Prolonged PR interval

Rationale: ST segment elevation is seen in variant (Prinzmetal's) angina. However,


sometimes ST elevation can be seen during pain with unstable angina as well.
Nonetheless, ST segment elevation is a cardinal clue to variant angina. Angina is
characterized by pain at rest, unrelated to exertion. ST segment depression is
characteristic of classic angina; although, it can also sometimes be seen with variant
angina. There is not an increase in heart rate seen with variant angina. Q waves are
seen with myocardial infarction. Prolonged PR interval is seen with first degree heart
block. References: 1. Andreoli, Tomas E., M.D., Carpenter, Charles, C.J., M.D., Griggs,
Robert C., M.D., Benjamin, Ivor J., M.D. Andreoli and Carpenter's Cecil Essentials of
Medicine. 7th edition. Philadelphia, PA. 2007. 2. M. Bory, F. Pierron, D. Panagides, J. L.
Bonnet, S. Yvorra and L. Desfossez. Coronary artery spasm in patients with normal or
near normal coronary arteries. Long-term follow-up of 277 patients. European Heart
Journal (1996) 17, 1015-1021.

5.A 33-year-old woman presents to the clinic stating that for the last couple of days she
feels that she is having an 'out of body experience' but still has 'a grip on reality.' Her
history reveals previous episodes similar to the one she is having today. She has denied
any drug or alcohol use. During her childhood she was molested by an uncle and was in
an abusive relationship during her early 20s. She states that she does not handle stress
well and believes she has had panic attacks in her past. This episode began after her
boss threatened to fire her over her constant tardiness. Her physical exam is
unremarkable. What is the most appropriate diagnosis for this patient?
a) Anxiety and depression
b) Schizophrenia
c) Substance abuse
d) Temporal lobe epilepsy
e) Depersonalization disorder

Rationale: Depersonalization disorder (DPD) typically occurs suddenly during periods of


anxiety where the individual feels that they are no longer in contact with their body or
surroundings. They typically describe the episode as an out of body experience or
feeling of living in a dream-like state, and many may feel that they are going crazy but
maintain a grip on reality. Depersonalization disorder may be elicited by anxiety,
depression, or borderline personality disorder. About 11-42% of patients diagnosed with
DPD have schizophrenia as a comorbid disease. Substance abuse can also elicit an
attack of DPD. In temporal lobe epilepsy, DPD can occur as an aura to the seizure, in
between seizures, or during the seizure itself and usually manifest by stereotypical
movements made by the epileptic (i.e. lip smacking). References: 1. Barry Nurcombe,
MD. Current Psychiatry > Section III. Syndromes and Treatments in Adult Psychiatry.
Chapter 27, Dissociative Disorders. 2. Simeon D, Knutelska M, Nelson D, Guralnik O.
Feeling unreal: a depersonalization disorder update of 117 cases. J Clin Psychiatry.
2003 Sep;64(9):990-7 3. Simeon D, Guralnik O, Schmeidler J, et al. The role of
childhood interpersonal trauma in depersonalization disorder. Am J Psychiatry. 2001
Jul;158(7):1027-33 4. Simeon D. Depersonalisation disorder: a contemporary
overview.CNS Drugs. 2004;18(6):343-54 5. HalesRE, Yudofsky SC. The American
Psychiatric Publishing Textbook of Clinical Psychiatry. American Psychiatric Pub , Inc.
2003:720 6. Sierra M, Berrios GE. The Cambridge Depersonalization Scale: a new
instrument for the measurement of depersonalization.Psychiatry Res. 2000 Mar
6;93(2):153-64. 7. Fauman MA. Study guide to DSM-IV-TR. American Psychiatric Pub,
Inc. 2002:282

6.An 18-year-old woman presents with shortness of breath, cough, malaise, and
dyspnea on exertion. Her symptoms began abruptly approximately 2 hours ago. She has
no history of illness or disease and has been very healthy up until this point. Physical
exam revealed a thin, ill-appearing young woman in obvious respiratory distress. Her
oxygen saturation was 82% and she had decreased breath sounds bilaterally. Vital signs
are as follows: Blood pressure 128/88 mm Hg, Pulse 124/min, Respiratory Rate 26 per
minute, Temp 97.8 degrees Fahrenheit. Spontaneous pneumothorax is suspected in the
patient. Which of the following concerning the patient’s probable diagnosis is true?
a) Women are affected more often than men
b) Peak incidence is between 40 and 50 years
c) They are usually the result of trauma
d) Smoking can increase the chance of developing this condition
e) The trapped air is fixed and will not shift to the highest point when the patient is
moved

Rationale: There is an association between smoking and the development of a primary


spontaneous pneumothorax. A large majority of patients are tall, thin young males with a
history of smoking. Primary spontaneous pneumothorax is a condition that occurs when
air is introduced into the pleural space without trauma, disease, or surgery as the cause.
They may be unilateral or bilateral. It may develop at rest and usually occurs in tall, thin
males with a high incidence between the ages of 20 and 30 years. A large percentage of
patients are smokers. It may resolve spontaneously if it is small or it may require the
insertion of a chest tube to re-inflate the lung(s). There is a risk of causing pulmonary
edema when the area is suctioned. This should be done with caution, limiting the
amount of force used to remove the air. Symptoms may include chest pain, shortness of
breath, dyspnea on exertion, anxiety, malaise, tachypnea, tachycardia, and decreased
breath sounds on the affected side. Air is not fixed within the area and will move to the
highest point when the patient’s position is changed. References: 1. Recurrence of
primary spontaneous pneumothorax, Thorax. 1997 Sep;52(9):805-809 2. Schwartz's
Principles of Surgery, 8th Edition, Chapter 18 3. Current Medical Diagnosis and
Treatment, 45th Edition, Chapter 9 4. Huang TW, Lee SC, Cheng YL,et al.Contralateral
Recurrence of Primary Spontaneous Pneumothorax. Chest. 2007 Jun 5 5. Olsen CM.
Spontaneous Pneumothorax Treatment Guidelines Previewed.2000;5:1

7.A 21-year-old woman presents at 10 weeks gestation with vaginal bleeding and lower
abdominal cramping. This is her second pregnancy and she is concerned about losing
her baby. Examination demonstrates bright red blood coming through a closed cervical
os. What is the most likely diagnosis?
a) Inevitable abortion
b) Complete abortion
c) Threatened abortion
d) Incomplete abortion
e) Habitual abortion

Rationale: The clinical picture is suggestive of a threatened abortion. About 75% of


abortions occur before the 16th week of gestation. In a threatened abortion, bleeding
and cramping occur but the cervix is not dilated. The pregnancy continues. In a complete
abortion, the placenta and fetus are completely expelled. In an inevitable abortion, the
cervix is dilated. In an incomplete abortion, mild cramps are reported but some of the
products of conception, usually the placenta, remain in the uterus. Habitual (recurrent)
abortion is defined as a loss of 3 or more previable pregnancies in succession.
Reference: 1. McPhee SJ, Papadakis MA, et al. Current Medical Diagnosis and
Treatment, McGraw-Hill, 2009

8.A 4-year-old girl presents you with her parents who witnessed her falling on an
outstretched right dominant arm with her elbow fully extended from a slide (about 4 feet
fall) 30 minutes ago. There was no loss of consciousness, but there was extreme pain
and she immediately grabbed her right forearm and her right elbow. There was an
obvious deformity at the elbow. Which of the following is the most likely diagnosis?
a) Proximal humerus fracture
b) Distal radius fracture
c) Supracondylar humerus fracture
d) Lateral epicondyle humerus fracture
e) Medial condyle humerus fracture

Rationale: Supracondylar fractures of the distal humerus are the most common elbow
fracture in children between the ages of 2 and 12. The typical mechanism of injury for
this type of fracture is a fall on an outstretched arm, usually from a height. Proximal
humerus and distal radius fractures won't show deformity at the elbow unless associated
with injury at the elbow too. Fracture of the lateral epicondyle of the humerus accounts
for 1/5th of elbow fractures in children. Fracture of the medial condyle of the humerus is
also uncommon. References: 1. Pizzutillo, P.D. Fractures about the Elbow. In: Griffin,
L.Y. (Ed.) Essentials of Musculoskeletal Care, 3rd ed. (2005), American Academy of
Orthopaedic Surgeons; pages 868-871. 2. Schatzker J. Tile M. The Rationale of
Operative Fracture Care. Birkhäuser. 2005:103 3. Lawrence P. Bell R. Dayton M.
Essentials of Surgical Specialties. Lippincott Williams& Wilkins. 2006:214 4. Chan O.
ABC of Emergency Radiology. Blackwell Publishing. 2007:27

9.A 34-year-old woman presents with redness and swelling of the joints of her hands.
She has noted that the symptoms are worse in the morning. On physical examination,
redness and swelling of her DIP and PIP joints on both hands are noted. Radiographic
imaging of her hands reveals joint erosion and space narrowing. Her sedimentation rate
(ESR) is elevated and her rheumatoid factor is negative. What is the most likely
diagnosis?
a) Osteoarthritis
b) Ankylosing spondylitis
c) Rheumatoid arthritis
d) Dupuytren’s contracture
e) Fibromyalgia

Rationale: The clinical picture is suggestive of rheumatoid arthritis (RA). RA


characteristically includes small joint symmetrical arthritis with an elevated ESR,
indicating an inflammatory process. Roughly 75-85% of patients have a positive
rheumatoid factor but 15-25% may not. Radiographic imaging includes juxta-articular
osteoporosis and joint erosion with space narrowing. Osetoarthritis would not have an
elevated ESR and radiological findings are not supportive of this diagnosis. Ankylosing
spondylitis would have an elevated ESR and negative rheumatoid factor, but mainly
involves the SI joint and lumbar/thoracic spine fusion. Dupuytren's contracture is
characterized by the hyperplasia of the palmar fascia with nodule formation and
contraction of the palmar fascia. Fibromyalgia is seen frequently in women ages 20-50
years of age but with widespread musculoskeletal pain with multiple tender points with
signs of inflammation absent. References: 1. McPhee SJ, Papadakis MA, et al. Current
Medical Diagnosis and Treatment, McGraw-Hill, 2008 2. Weinstein SL, Turek SL,
Buckwalter JA. Turek's Orthopaedics. Lippincott Williams & Wilkins, 2005:163 3. Stoller
DW. Magnetic Resonance Imaging in Orthopaedics and Sports Medicine. Lippincott
Williams & Wilkins, 2006:652

10.A 24-year-old man comes to your clinic with a complaint of radial-sided non-dominant
wrist pain following a fall on his outstretched hand 3 days ago. He didn’t seek
immediate medical attention “because there was no swelling”. Despite self
treatment of ice and analgesics, his wrist has continued to hurt, especially with ulnar
deviation and trying to open a jar or grip the steering wheel of his car. You examine the
patient and determine his tenderness is in the anatomic snuffbox of the wrist.
Radiographs are normal in all views. Which of the following is the proper treatment at
this time for this man?
a) Elastic bandage for support, continue ice and analgesics, follow up as needed
b) Wrist volar splint, continue ice and analgesics, follow up in a month
c) Thumb spica splint, limit use, follow up in 2 to 3 weeks for repeat
radiographs
d) Immediate referral to an orthopedic surgeon
e) Routine referral to an orthopedic surgeon

Rationale: If a patient with a suspected scaphoid fracture has radiographs that are read
as "normal", but clinically there is a fracture, place the patient in a long arm thumb spica
splint for 2 to 3 weeks, then repeat the radiographs. Elastic bandage and volar splints
are not adequate immobilization for a possible scaphoid fracture. Immediate referral is
not warranted because you have not identified a positive diagnosis, and routine referral
is not warranted for the same reason. However, if after a positive diagnosis AND
evidence of non-union OR worsening fracture gap, a referral to an orthopedic surgeon is
definitely warranted at that point. References: 1. Seiler III, J.G. Fracture of the Scaphoid.
In: Griffin LY, ed. Essentials of Musculoskeletal Care, 3rd ed. Rosemont, IL: American
Academy of Orthopaedic Surgeons; 2005:362. 2. Cooney WP 3rd. Scaphoid fractures:
current treatments and techniques. Instr Course Lect. 2003;52:197-208 3. Lewis C.
General Orthopaedics. McGraw-Hill Professional. 2003:118

11.A 60-year-old female has been diagnosed by you as having major depression. You
have completed the history, physical, and lab testing and started the patient on
antidepressant medication. You also feel she would benefit from talking to someone
about some of stressful events that have occurred in the past. Which of the following
therapies would you consider for the same?
a) Family therapy
b) Brief interpersonal psychotherapy
c) Group therapy
d) Behavior therapy
e) Cognitive therapy

Rationale: There are several indicators of major depression. A major depressive


disorder (MDD) must consist of several symptoms for at least 2 weeks. These are
depressed mood, decreased interest in normal activities, significant weight loss or gain,
insomnia or hyper-sleeping, psychomotor agitation or retardation, feelings of
worthlessness or guilt, and/or poor concentration and possible passive or direct suicidal
threats. Depressed mood is an indicator, but not a diagnosis. Treatment of major
depression in older adults is most successful with a combination of antidepressant
medications plus interpersonal supportive psychotherapy. Interpersonal psychotherapy
is suitable for individuals having experienced recent interpersonal conflicts or difficult
role transitions. The focus in interpersonal therapy is on role disputes and transitions,
personal losses, deficits in social skills, social isolation, and other interpersonal factors
that contribute to the development of depression. The approaches in interpersonal
therapy help to resolve role disputes and transitions, facilitate mourning and promote
recognition of related affects, and also help to overcome social skill deficits. Hence,
interpersonal therapy is more applicable to this patient. Studies have shown that brief
interpersonal psychotherapy is as effective as antidepressants for mild to moderately
severe major depressive disorder. Patients with MDD often have irrational beliefs and
distorted attitudes toward the self and the environment. Cognitive therapy aims at
providing structured guidance to correct their distorted concepts of themselves and
others. Family problems increase the vulnerability to MDD and may also retard recovery,
hence family therapy is conducted for a comprehensive treatment modality. Group
therapy encompasses psychotherapy conducted in groups. Certain psychotherapies
such as behavior and cognitive therapy can be conducted in groups. Behavioral therapy
aims at changing the negative styles of thinking and behaving associated with
depression. Behavioral, cognitive, group, and family therapies have shown less
promising results. Also, brief or short therapy has shown to be as effective or more
effective as long term psychotherapy. Reference: 1. Swartz HA, Frank E, Shear MK,
Thase ME, Fleming MA, Scott J. A pilot study of brief interpersonal psychotherapy for
depression among women. Psychiatr Serv. 2004; 55(4):448-50. 2. Badamgarav E,
Weingarten SR, Henning JM et al. Effectiveness of disease management programs in
depression: A systematic review. Am J Psychiatry 2003; 160(12):2080-2090. 3. NHS
Centre for Reviews and Dissemination. Improving the recognition and management of
depression in primary care. Effective Health Care 2002; 7(5):1-12.

12.A 45-year-old man presents to the hospital complaining of a 1-month history of


difficulties walking up the stairs to his apartment. He also reports that the joints of his
hands have been paining. He is not taking any medications. On examination, his BP is
120/80mm/Hg and PR 72/min. Laboratory investigations reveal positive anti-nuclear
antibodies. Muscle biopsy reveals atrophy of type II muscle fibers. Question: What is the
most likely cause of his muscle weakness?
a) Alcoholism
b) Hyperthyroidism
c) Cushing's disease
d) Hypothyroidism
e) Systemic Lupus Erythematosus

Rationale: Muscle weakness due to systemic lupus erythematosus (SLE) affects the
proximal muscle groups of the shoulder and pelvic girdles. Patients have difficulties
doing everyday activities, such as walking. In addition to the weakness, they may also
present with other signs and symptoms of SLE, such as a malar rash and joint pains.
Laboratory results will reveal positive anti-nuclear antibodies (ANA) and depressed C3
complement levels. Muscle biopsies reveal atrophy of type II muscle fibers. Muscle
weakness due to alcoholism affects the proximal muscles, though the distal muscles
may also be affected. Patients complain of difficulties doing everyday activities, such as
combing their hair. They may also complain of tingling sensations due to peripheral
neuropathy. On examination, they may have telangiectasia and other signs of chronic
alcohol abuse. Laboratory investigations may reveal low hemoglobin and vitamin B12
levels and elevated transaminase and gamma GT (GGT) levels. The electromyogram
will be normal. A muscle biopsy shows nonspecific myopathic changes, such as atrophy
and regeneration of muscle fibers. Muscle weakness due to hyperthyroidism affects the
proximal and bulbar muscles. Patients are usually in their forties. They have other
symptoms of hyperthyroidism, such as excessive perspiration, emotional lability, and
weight loss. On examination, they may have tachycardia, exophthalmos, and fine
tremors. Laboratory results reveal elevated T3 and T4 levels. Electromyogram reveals
myopathic motor unit action potentials. Muscle biopsies are usually normal. Muscle
weakness due to hypothyroidism affects the proximal muscle groups. Patients complain
of difficulties doing everyday activities, such as combing their hair. It usually affects
patients in the 30 to 49-year age group. They may also have other symptoms of
hypothyroidism, such as menorrhagia and weight gain. On examination, they may have
a goiter and bradycardia. Laboratory results reveal elevated thyroid stimulating hormone
(TSH) and creatine kinase. Muscle biopsies reveal glycogen accumulation and
nonspecific myopathic changes, such as atrophy and regeneration of muscle fibers.
Muscle weakness due to Cushing's disease affects the proximal muscle groups. Patients
complain of difficulties in everyday activities, such as rising from a chair. They may have
other symptoms associated with glucocorticoid excess, such as easy bruising and
emotional lability. On examination, they may have truncal obesity, the characteristic
buffalo hump, and striae. Laboratory results reveal a normal creatine kinase and
elevated free cortisol in the urine. Electromyogram shows myopathic motor unit action
potentials. Muscle biopsies show selective atrophy of type II muscle fibers. References:
1. The Merck Manual 2. Harrison's Principles of Internal Medicine

13.When a rectovaginal exam is performed, the body of the uterus is palpated. This
finding indicates that the uterus is in what type of position?
a) An anteverted uterus
b) A retroflexed uterus
c) A retroverted uterus
d) A second-degree prolapsed uterus
e) A third-degree prolapsed uterus

Rationale: ?A uterus that cannot be palpated by a bimanual exam but can be felt
rectovaginally is tilted backwards. A retrodisplaced uterus wherein the cervix is normally
positioned, as in this case, is in a retroflexed position. A retrodisplaced uterus with a
forward-facing cervix is a uterus that is in a retroverted position. An anteverted uterus is
a forward-facing uterus, the most common position found. In a second-degree prolapse,
the cervix is in the vaginal introitus with the body of the uterus descending from its
normal placement in the pelvic. In a third-degree prolapsed, the cervix and uterus are
outside the vaginal introitus. Reference: 1. Bickley L, Szilagyi P. Bate's Guide to Physical
Examination. 8th ed. Philadelphia, PA: Lippincott, Wilkins and Williams; 2003:407.
14.A 67-year-old man was cleaning out his garage and noticed an insect crawling on his
leg. The insect bit him and was then killed by the man. He discarded it and went about
his business. 2 days later, he went to the Emergency Department with complaints of
pain, itching, and swelling of the affected leg. The insect had a violin-shaped pattern on
its back. What was the insect responsible?
a) Scorpion
b) Black widow spider
c) Wasp
d) Mosquito
e) Brown recluse spider

Rationale: The brown recluse spider is one of only about 4 spiders capable of being
dangerous to humans. They are native to the Midwestern and Southeastern sections of
the United States. The bite from one can cause death if treatment is not given. The
brown recluse spider can be recognized by the "violin" pattern found on its back. They
are usually dark brown in color and have dark brown to black colored legs. They usually
do not attack unless frightened or when they touch the skin of an animal or human. They
can be found in closets, basements, sleeping bags, woodpiles, or other warm, dry
places. The venom of a brown recluse spider is extremely poisonous and its toxin
causes damage to cells and tissues. This is done through the use of enzymes that can
destroy cell membranes. This can lead to the local destruction of the skin. It can also
spread to the blood vessels and fat. The skin becomes necrotic in the area of the bite,
which is characteristic for this spider. The immune system becomes activated as with
other insect bites. Brown recluse spider bites can lead to necrosis of the skin,
destruction of red blood cells, blood clot formation, acute renal failure, coma, and death.
Symptoms of a brown recluse spider bite can be similar to a bee sting. There can be
mild erythema, pain, itching, and myalgias. These symptoms usually develop within a
few hours. Within a few days, the area will heal. In severe cases the patient may develop
fever, nausea, vomiting, blistering, and eventually necrosis of the affected skin. The
diagnosis can be made through the patient's history and by examining the affected area.
Laboratory analysis should include a complete blood count, urinalysis, electrolyte, and
renal function tests. Usually no specific tests are done. Treatment usually consists of:
tetanus immunization, antihistamines, and antibiotics. In some cases, steroid treatments
are used. This is usually reserved for patients who have an underlying illness such as
anemia or renal failure. References: 1. Current Diagnosis & Treatment: Emergency
Medicine, 6th Edition, Chapter 44. 2. Tintinalli's Emergency Medicine: A Comprehensive
Study Guide, 6th Edition, Chapter 194

15.A 28-year-old woman came to the ER presenting with vaginal bleeding, abdominal
pain, and light headedness. The patient said that pain started 5 hours ago, sharp and
increasing in severity, then bleeding followed. Her last menstrual period was 76 days
ago and she used to have regular menses every 31 days. She is sexually active with
multiple partners. She was treated several times for STD. On examination, the patient is
hypotensive, and on abdominal examination there is tenderness and rebound;
tenderness especially at right hypochondrial area. What is the most likely diagnosis?
a) Acute appendicitis
b) Ruptured ectopic pregnancy
c) Threatened abortion
d) Endometriosis
e) Inevitable abortion

Rationale: Ectopic pregnancy should be suspected in any woman with amenorrhea


presenting with abdominal pain and vaginal bleeding. Its serious and potential
complications include hypovolemia and even death due to shock. Ultrasound helps in
diagnosis especially transvaginal ultrasound. Acute appendicitis has similar symptoms
without vaginal bleeding. Threatened or inevitable abortion is generally not associated
with acute abdomen. Endometriosis is associated with vaginal bleeding and pain, but
usually irregular menses and no acute abdominal pain. References: 1. Richard S.
Krause, David M. Janicke, Tintinalli's Emergency Medicine (6),Section 11: Gynecology
and Obstetrics , Chapter 103. Ectopic Pregnancy. 2. Wong E, Suat SO. Ectopic
pregnancy--a diagnostic challenge in the emergency department. Eur J Emerg Med.
2000 Sep;7(3):189-94 3. Sinaii N, Plumb K, Cotton L. Differences in characteristics
among 1,000 women with endometriosis based on extent of disease. Fertil Steril. 2008
Mar;89(3):538-45. Epub 2007 May 11

16.A 45-year-old man presents to your office complaining of a 2-month history of


weakness of his shoulder muscles. He reports he has been gaining a lot of weight,
though he has not changed his diet. He drinks alcohol everyday. On examination, his BP
is 145/85 mm/Hg and PR 95/min. He is obese but not pale or jaundiced. Laboratory
investigations reveal normal transaminases and creatine kinase. A muscle biopsy shows
selective atrophy of type II muscle fibers. What is the most likely cause of his muscle
weakness?
a) Alcoholism
b) Hyperthyroidism
c) Cushing's disease
d) Hypothyroidism
e) Systemic Lupus Erythematosus

Rationale: ?Muscle weakness due to Cushing's disease affects the proximal muscle
groups. Patients complain of difficulties in everyday activities, such as rising from a
chair. They may have other symptoms associated with glucocorticoid excess, such as
easy bruising and emotional lability. On examination, they may have truncal obesity, the
characteristic buffalo hump, and striae. Laboratory results reveal a normal creatine
kinase and elevated free cortisol in the urine. Electromyogram shows myopathic motor
unit action potentials. Muscle biopsies show selective atrophy of type II muscle fibers.
Muscle weakness due to alcoholism affects the proximal muscles, though the distal
muscles may also be affected. Patients complain of difficulties doing everyday activities,
such as combing their hair. They may also complain of tingling sensations due to
peripheral neuropathy. On examination, they may have telangiectasia and other signs of
chronic alcohol abuse. Laboratory investigations may reveal low hemoglobin and vitamin
B12 levels and elevated transaminase and gamma GT (GGT) levels. The
electromyogram is normal. A muscle biopsy shows nonspecific myopathic changes,
such as atrophy and regeneration of muscle fibres. Muscle weakness due to
hyperthyroidism affects the proximal and bulbar muscles. Patients are usually in their
forties. They have other symptoms of hyperthyroidism, such as excessive perspiration,
emotional lability, and weight loss. On examination, they may have tachycardia,
exophthalmos, and fine tremors. Laboratory results will reveal elevated T3 and T4 levels.
Electromyogram reveals myopathic motor unit action potentials. Muscle biopsies are
usually normal. Muscle weakness due to hypothyroidism affects the proximal muscle
groups. Patients complain of difficulties doing everyday activities, such as combing their
hair. It usually affects patients in the 30 to 49-year age group. They may also have other
symptoms of hypothyroidism, such as menorrhagia and weight gain. On examination,
they may have a goiter and bradycardia. Laboratory results will reveal elevated thyroid
stimulating hormone (TSH) and creatine kinase. Muscle biopsies reveal glycogen
accumulation and nonspecifice myopathic changes, such as atrophy and regeneration of
muscle fibers. Muscle weakness due to systemic lupus erythematosus (SLE) affects the
proximal muscle groups of the shoulder and pelvic girdles. Patients have difficulties
doing everyday activities, such as walking. In addition to the weakness, they may also
present with other signs and symptoms of SLE, such as a malar rash and joint pains.
Laboratory results will reveal positive anti-nuclear antibodies (ANA) and depressed C3
complement levels. Muscle biopsies reveal atrophy of type II muscle fibers. References:
1. The Merck Manual 2. Harrison's Principles of Internal Medicine

17.A 16-year-old boy states that he was camping with his family in Arizona and was
bitten by an insect that he did not see. The wound was cleaned and the boy's camping
trip continued without any further interruption. Approximately 5 hours after being bitten,
he began feeling intense pain, itching, fever, and nausea. The affected area was red
initially, but then became blistered and bluish in color. At the time of presentation the
area showed signs of necrosis. What is the insect that most likely caused this patient's
symptoms?
a) Brown recluse spider
b) Wasp
c) Hornet
d) Black Widow spider
e) Yellow Jacket

Rationale: The brown recluse spider is one of only about 4 spiders capable of being
dangerous to humans. They are native to the Midwestern and Southeastern sections of
the United States. The bite from one can cause death if treatment is not given. The
brown recluse spider can be recognized by the "violin" pattern found on its back. They
are usually dark brown in color and have dark brown to black-colored legs. They usually
do not attack unless frightened or when they touch the skin of an animal or human. They
can be found in closets, basements, sleeping bags, woodpiles, or other warm, dry
places. The venom of a brown recluse spider is extremely poisonous and its toxin
causes damage to cells and tissues. This is done through the use of enzymes that can
destroy cell membranes. This can lead to the local destruction of the skin. It can also
spread to the blood vessels and fat. The skin becomes necrotic in the area of the bite,
which is characteristic for this spider. The immune system also becomes activated, as
with other insect bites. Brown recluse spider bites can lead to necrosis of the skin,
destruction of red blood cells, blood clot formation, acute renal failure, coma, and death.
Symptoms of a brown recluse spider bite can be similar to a bee sting. There can be
mild erythema, pain, itching, and myalgias. These symptoms usually develop within a
few hours. Within a few days, the area will heal. In severe cases the patient may develop
fever, nausea, vomiting, blistering, and eventually necrosis of the affected skin. The
diagnosis can be made through the patient's history and by examining the affected area.
Laboratory analysis should include a complete blood count, urinalysis, electrolyte, and
renal function tests. Usually no specific tests are done. Treatment usually consists of
tetanus immunization, antihistamines, and antibiotics. In some cases, steroid treatments
are used. This is usually reserved for patients who have an underlying illness such as
anemia or renal failure. References: 1. Current Diagnosis & Treatment: Emergency
Medicine, 6th Edition, Chapter 44. 2. Tintinalli's Emergency Medicine: A Comprehensive
Study Guide, 6th Edition, Chapter 194.

18.A 51-year-old male with a history of Marfan syndrome presents to the emergency
department with tearing chest pain that radiates to the back and neck. On examination,
the patient is hypertensive, and a high-pitched decrescendo diastolic murmur at the left
sternal border and diminished peripheral pulses are noted. Based on the patient’s
most likely diagnosis, what is the immediate diagnostic imaging modality of choice?
a) Posterior-anterior chest x-ray (PA CXR)
b) Computed tomography scan of the chest and abdomen (CT scan)
c) Magnetic resonance imaging of the chest (MRI)
d) 12-lead Electrocardiogram (ECG)
e) Transesophageal echocardiography (TEE)

Rationale: The patient in the above scenario most likely has an acute aortic dissection.
Aortic dissections occur when a tear develops in the intimal layer of the vessel, allowing
blood to collect in the medial layer. These tears can be the result of the repetitive torque
applied to the ascending and proximal descending aorta during the cardiac cycle,
especially in patients with hypertension. Abnormalities of smooth muscle, elastic tissue,
or collagen, as seen in Marfan syndrome, can also predispose vessels to dissection.
There are 2 types of aortic dissection. Type A dissection involves the arch proximal to
the left subclavian artery. Type B dissection typically occurs in the proximal descending
thoracic aorta, just distal to the left subclavian artery. A CT scan is the immediate
diagnostic imaging modality of choice for patients with suspected aortic dissection. The
CT should include both the chest and the abdomen so that the extent of the dissection
can be fully delineated. Chest radiographs may reveal an abnormal aortic contour or a
widened mediastinum to suggest dilation or other pathologic process of the aorta but are
not specific enough to make a diagnosis of an aortic dissection. An MRI is an excellent
imaging modality for chronic dissections. However, the longer imaging time and the
difficulty of monitoring patients in the MRI scanner make the CT preferable in the acute
setting. Evidence of left ventricular hypertrophy from chronic hypertension can be noted
on an ECG. Acute ST changes, suggesting myocardial ischemia can develop if the
dissection involves the coronary artery ostium. Of these, inferior wall abnormalities are
the most common since dissection leads to compromise of the right coronary artery
rather than the left. It some patients with an aortic dissection the ECG may be
completely normal. Transesophageal echocardiograms are also excellent in diagnosing
aortic dissections. However, because this type of echocardiography is more clinically
invasive, it is generally not readily available in the acute setting. Reference: 1. Rapp JH,
Sarkar R, Chang CK. Blood Vessels & Lymphatics. In: McPhee SJ, Papadakis MA,
Tierney LM, eds. Current Medical Diagnosis & Treatment. 47th ed. New York, NY:
McGraw-Hill; 2008: 398-421.

19.In reviewing your hospital's policies, you notice the presence of an Institutional Ethics
Committee that can provide ethics consultations. The provision of an ethics consultation
by an Institutional Ethics Committee is best represented by the statement that:
a) Any patient, parent, guardian, or family member should be able to initiate
an ethics consultation
b) Refusal of a patient, parent, or guardian to permit an ethics consultation should
be respected and honored
c) Only the physician directly involved in the care of the patient should be able to
request an ethics consultation without fear of reprisal
d) Anonymous requests for consultation should only be accepted in the absence of
an identified person willing to speak on the issue raised
e) The process of consultation should be open and readily accessible to all persons
involved in the patient's care
Rationale: Institutional ethics committees (IECs) throughout hospitals in the United
States serve to educate health professionals about biomedical ethics, draft and review
hospital policy, and initiate clinical ethics case consultations. IECs are an important
mechanism for the discussion and resolution of ethical issues raised in the provision of
patient care. A "consultation team" may be requested to review a clinical situation where
conflicts about the value of interventions arises, when communication breaks down, or
when there is uncertainty about how to respond when questions of moral, legal, or
economic justification are raised, such that patient care may be impeded. The
involvement of a patient, family, or both, and the documentation in medical records
distinguish an ethics consultation from an informal request for advice. AN IEC should
have policies and procedures to determine who can request a consultation, how the IEC
is contacted, who responds to the request, how the consultation is conducted, who is to
be included, what constitutes proper notification, what protects patient confidentiality and
documentation, when ethics consultation is required, and what defines the advisory
nature of the recommendations. Acceptable approaches to implementing this vary from
case to case; however, the following guidelines should always apply to ensure fairness
and accountability: 1. Any patient, parent, guardian, or family member should be able to
initiate an ethics consultation. 2. The patient and parent or guardian should be able to
refuse to participate in an ethics consultation. 3. The refusal of a patient or parent or
guardian to permit an ethics consultation should not obstruct the ability of an ethics
committee to provide consultation services to physicians, nurses, and other concerned
staff. 4. Any physician, nurse, or other health care provider who is involved in the care of
the patient should be able to request an ethics consultation without fear of reprisal. 5.
The process of consultation should be open to all persons involved in the patient's care,
yet conducted in a manner that respects patient and family confidentiality. 6. Anonymous
requests for consultation should not be accepted in the absence of an identified person
who is willing to speak to the issue being raised. 7. The primary care pediatrician should
be invited to participate in the consultation to support existing physician-family
relationships References: 1. American Academy of Pediatrics Committee on Bioethics.
Institutional ethics committees. Pediatrics. 2001;107:205-209 2. Mark R. Mercurio,
Edwin N. Forman, Rosalind Ekman Ladd, Marilyn A. Maxwell, Lainie Friedman Ross,
and Tomas J. Silber American Academy of Pediatrics Policy Statements on Bioethics:
Summaries and Commentaries: Part 3 Pediatr. Rev., May 2008; 29: e28 - e34. 3. van
der Kloot Meijburg HH, ter Meulen RH. Developing standards for institutional ethics
committees: lessons from The Netherlands.J Med Ethics. 2001 Apr;27 Suppl 1:i36-40 4.
American Academy of Pediatrics. Committee on Bioethics. Institutional ethics
committees. Committee on Bioethics. Pediatrics. 2001 Jan;107(1):205-9.

20.A 38-year-old woman presents to your clinic with the complaint of a tender mass on
the dorsum of her right dominant wrist for the past 3 months. It has fluctuated in size and
even gone away for a while. She works in a factory where she uses her hands
extensively, and now the mass is bothering her enough to cause her to have trouble
doing her job. The doctor tells you that this patient has a dorsal wrist ganglion cyst.
Which of the following treatment options would you offer first in this patient?
a) AROM
b) PROM
c) Observation
d) Immobilization & activity modification

Rationale: Because this patient is having problems with performing her job and is
having some discomfort with the mass, a trial of immobilization and activity modification
should be done first. Although observation should be considered, it seems that she has
already undergone that period of observation with it "coming and going". Needle
aspiration may also be considered, but has possible complications that immobilization
and activity modification would not have. The ganglion cyst used to be called the "Bible
cyst" and reportedly was smashed by a Baptist minister's Bible and the devil was
"exorcised" from the parishioner. This treatment method is not used today. As the
recurrence rate after aspiration and surgical excision is high, surgical excision is the final
and not the initial choice of treatment. References: 1. Seiler III, JG. Ganglia of the Wrist
and Hand. In: Griffin LY, ed. Essentials of Musculoskeletal Care, 3rd ed. Rosemont, IL:
American Academy of Orthopaedic Surgeons; 2005:362 2. Dias JJ, Dhukaram V, Kumar
P. The natural history of untreated dorsal wrist ganglia and patient reported outcome 6
years after intervention. J Hand Surg Eur Vol. 2007 Oct;32(5):502-8. Epub 2007 Aug 6
3. Seiler J. Essentials of Hand Surgery. Lippincott Williams& Wilkins. 2002:167 4.
Nielsen NH, Jensen NV. [Ganglion--cysts of the hand and wrist] [Article in Danish]
Ugeskr Laeger. 2007 Apr 2;169(14):1303-5.

21.A 45-year-old woman came to the outpatient clinic presenting with fatigue. On review
of her symptoms, she admitted that she has a good appetite and no cold intolerance.
She denied feeling depressed or having any pain. Her bowel habit is once a day, and
she is 240 pounds, 5 feet 4 inches tall. She has no urinary troubles. She gave history of
snoring at night and repetitive awakening in the middle of night, as reported by her
husband. What is the most likely diagnosis?
a) Narcolepsy
b) Sleep apnea
c) Hidden malignancy
d) Diabetes mellitus
e) Hypothyroidism

Rationale: Obstructive sleep apnea is one of the most important diseases that have
been diagnosed in the last 50 years. It has high morbidity and mortality. It requires
lifelong treatment. So, detailed assessment is required including detailed physical
examination, thyroid functions screening, and polysomnography. It is a condition
characterized by cessation of breathing (apnea) for at least 10 seconds during sleep,
usually witnessed by another person as the patient struggles to breathe. The patient is
usually obese and hypertensive. It happens due to loss of pharyngeal wall tone leading
to narrowing of airways and obstruction of upper respiratory passages. Patients with
narrowed airways to begin with are more prone to these attacks. The symptoms are day-
time sleepiness and fatigue. Alcohol and hypnotic drugs worsen the case by decreasing
pharyngeal wall tone. Sleep study is an essential step in the diagnosis by finding the
apneic spells. Continuous positive airway pressure breathing (CPAP) is helpful to
prevent pulmonary hypertension and cor pulmonale if used early. Surgery may be
required. Soft palate excision, uvulectomy or even tracheostomy may be necessary in
severe cases. Narcolepsy is sudden attacks of sleep at anytime even during activities
like classes or work. Hidden malignancy can be a cause of fatigue, but usually
associated with other symptoms such as masses, bleeding, anemia, or change in urine
or bowel habit. Hypothyroidism is generally associated with constipation, cold
intolerance, fatigue, and weight gain. Diabetes mellitus is characterized by fatigue,
polyuria, polydipsia, and weight gain or loss, according to whether it is insulin dependent
(loss) or not (gain). It is sometimes associated with sleep apnea; in fact, sleep apnea
increases insulin resistance in diabetic patients and its treatment decreases insulin
requirements. References: 1. Neil J. Douglas, Chapter 259. Sleep Apnea , Section 2:
Diseases of the Respiratory System in Harrison's Online, Part Ten: Disorders of the
Respiratory System 17th edition 2. Kevin Welch & Andrew Goldberg, Sleep Apnea,
Chapter 40. Sleep Disorders, Sleep Disorders in Adults in CURRENT Otolaryngology
2nd edition 3. Neil J. Douglas, Chapter 259. Sleep Apnea in Harrison's Online, Part Ten:
Disorders of the Respiratory System, Section 2: Diseases of the Respiratory System. ©
The McGraw-Hill Companies 4. Tasali E, Ip MS. Obstructive sleep apnea and metabolic
syndrome: alterations in glucose metabolism and inflammation. Proc Am Thorac Soc.
2008 Feb 15;5(2):207-17 5. Punjabi NM. The epidemiology of adult obstructive sleep
apnea.Proc Am Thorac Soc. 2008 Feb 15;5(2):136-43 6. Bordow RA, Ries AL, Morris
TA. Manual of Clinical Problems in Pulmonary Medicine. Lippincott Williams & Wilkins.
2005:442

22.A 62-year-old woman presents to the Emergency Department with right-sided chest
pain and shortness of breath. Her symptoms began abruptly about 2 hours ago. Her
history is significant for a 30-year smoking history, but she has been relatively healthy
otherwise. Examination reveals a thin, frail, ill-appearing woman in respiratory distress.
She was tachypneic with decreased breath sounds on the right. A chest radiograph
showed a large right-sided pneumothorax. Vital signs are as follows: Blood pressure
100/66 mm Hg, Pulse 126/min, Respiratory Rate 24 per minute, Temp 97.2 degrees
Fahrenheit. Which of the following concerning a secondary spontaneous pneumothorax
is true?
a) There is an increased risk in smokers
b) Recurrence rate is about 80%
c) They are always visible on an inspiratory chest radiograph
d) They are usually idiopathic and do not necessarily represent an underlying
disease process
e) It occurs more often in women

Rationale: There is an increased risk of pneumothorax with smoking. Secondary


spontaneous pneumothorax occurs as the result of some underlying disease presence.
It is often seen when the patient has COPD and it occurs mainly in older patients. A
large percentage of patients are smokers. They may resolve spontaneously or may
require the insertion of a chest tube to re-inflate the lung(s). There is a risk of causing
pulmonary edema when the area is suctioned. Therefore, this should be done with
caution, limiting the amount of force used to remove the air. The rate of recurrence is
variable and can be anywhere between 5 and 60%. The recurrence rate will be greater if
the underlying disease has not been resolved. A chest radiograph may not always show
the gas collection on an inspirational film. However, it is usually visible with expiration.
Treatment usually involves the placement of a chest tube and treatment of the
underlying disease process. References: 1. Recurrence of primary spontaneous
pneumothorax, Thorax. 1997 Sep;52(9):805-809 2. Schwartz's Principles of Surgery, 8th
Edition, Chapter 18 3. Current Medical Diagnosis and Treatment, 45thEdition, Chapter 9
4. Gotsman I, Goral A, Nusair S. Secondary spontaneous pneumothorax in a patient
with pulmonary rheumatoid nodules during treatment with methotrexate. Rheumatology
(Oxford). 2001 Mar;40(3):350-1 5. Huang TW, Lee SC, Cheng YL,et al.Contralateral
Recurrence of Primary Spontaneous Pneumothorax. Chest. 2007 Jun 5

23.A 56-year-old woman came to the outpatient clinic presenting with lightheadedness
for 1 week. She experiences it usually when she gets up from her bed. She is
hypertensive. She gave a 3-day history of a diarrhea. She reported eating some old
food. She doesn't have any hearing problems, ear pain, tinnitus, or discharge. She
denies any headache, vomiting, abnormal weakness, or sensations. You suspect this
patient has:
a) Stroke
b) Transient ischemic attack (TIA)
c) Meniere's disease
d) Orthostatic hypotension
e) Benign positional vertigo

Rationale: This patient has the typical presentation of orthostatic hypotension. The
symptom that gives that specific diagnosis is lightheadedness on getting up. The history
of diuretic therapy plus recent diarrhea and fluid loss further supports the hypovolemic
state that caused the patient's problem. Both TIA and stroke diagnoses require the
occurrence of weakness, paresthesia, or any other focal neurological deficit, or loss of
consciousness, seizure, impairment of memory, or change in cognition. Benign
positional vertigo is a disease of semicircular canals characterized by dizziness and
vertigo without hearing loss with certain movements of the head. It occurs suddenly and
lasts for a few seconds. Characteristic latent nystagmus occurs with Dix-Hallpike test.
Meniere's disease is usually associated with ear problems such as tinnitus, fullness, or
hearing deficit, together with dizziness. References: 1. Mark D. Carlson, orthostatic
hypotension, Chapter 21. Syncope In Harrison's Online Part Two: Cardinal
Manifestations and Presentation of Diseases, Section 3: Nervous System Dysfunction
17th edition 2. Jacob Johnson, & Anil Lalwani, Benign Paroxysmal Positional Vertigo In
current otolaryngology, XIII. Inner Ear, Chapter 56. Vestibular Disorders. 2nd edition.
The McGraw-Hill Companies. 3. Medow MS, Stewart JM, Sanyal S, et al.
Pathophysiology, diagnosis, and treatment of orthostatic hypotension and vasovagal
syncope. Cardiol Rev. 2008 Jan-Feb;16(1):4-20 4. Halker RB, Barrs DM, Wellik KE, et
al. Establishing a Diagnosis of Benign Paroxysmal Positional Vertigo Through the Dix-
Hallpike and Side-Lying Maneuvers: A Critically Appraised Topic.Neurologist. 2008
May;14(3):201-204 5. de Sousa LC, Piza MR, da Costa SS. Diagnosis of Meniere's
disease: routine and extended tests. Otolaryngol Clin North Am. 2002 Jun;35(3):547-64.
6. Balch PA. Prescription for Nutritional Healing . Avery. 2007:562 7. Freeman R.
Current pharmacologic treatment for orthostatic hypotension.Clin Auton Res. 2008
Mar;18 Suppl 1:14-8. Epub 2008 Mar 27

24.At the obstetric department, a baby who is totally blue. The baby is relieved when he
cries. He is a full term, average weight, vaginal delivery. There were no complications
throughout pregnancy or during birth. What is the most likely diagnosis?
a) Tracheoesophageal fistula
b) Meconium aspiration
c) Respiratory distress syndrome
d) Bilateral choanal Artesia
e) Central respiratory depression

Rationale: Choanal atresia is a congenital malformation characterized by obstruction


which may be membranous or bony at posterior nasal choanal. Unilateral atresia is
usually discovered late and usually confused with chronic sinusitis, but bilateral atresia
causes severe respiratory obstruction and should always be suspected in a new born
with respiratory obstruction alleviated by crying. Oral airway must be placed immediately
upon failure of a nasal tube to pass, or via confirmation by axial CT. Central respiratory
depression is not alleviated with crying. In respiratory distress syndrome where there is
non-complete development of lung surfactant, usually in premature babies, and in
meconium aspiration which usually occurs when there is prolonged labor, there is
persistent cyanosis and evident cause. Tracheoesophageal fistula - distress is usually
related to feeding, and not to the presence or absence of crying. References: 1. Peggy
E. Kelley et al., Current Pediatrics18th Edition, Chapter 17. Ear, Nose, & Throat 2.
Daniel SJ. The upper airway: congenital malformations. Paediatr Respir Rev. 2006;7
Suppl 1:S260-3. Epub 2006 Jun 6 3. Friedman NR, Mitchell RB, Bailey CM.
Management and outcome of choanal atresia correction. Int J Pediatr Otorhinolaryngol.
2000 Jan 30;52(1):45-51 4. Escande B, Kuhn P, Rivera S, Messer J. Secondary
surfactant deficiencies. Arch Pediatr. 2004 Nov;11(11):1351-9

25. 57-year-old Caucasian woman presents to your clinic complaining of a 2-week


history of feeling weak. She is an artist who has been chain smoking for 30 years. On
examination, she is wasted but not dehydrated. Her BP is 116/70 mmHg. Laboratory
investigations reveal plasma sodium of 122 mEq/L with a high urine osmolality. A chest
radiograph reveals a rounded opacity in the right lung field What is the most likely
diagnosis?
a) Lambert-Eaton myasthenic syndrome
b) Hypertrophic pulmonary osteoarthropathy
c) Horner’s syndrome
d) Ectopic adrenocorticotropic hormone (ACTH) secretion
e) Syndrome of inappropriate antidiuretic hormone secretion (SIADH)

Rationale: The syndrome of inappropriate antidiuretic hormone secretion (SIADH) is a


paraneoplastic syndrome associated with bronchogenic carcinoma. There is excessive
water reabsorption by the kidneys due to the elevated antidiuretic hormone (ADH).
Patients may complain of feeling weak and lethargic. Other symptoms include confusion
and coma. Laboratory tests reveal hyponatremia, high urine sodium, low serum
osmolality, and high urine osmolality in a euvolemic patient. In hypertrophic pulmonary
osteoarthropathy (HPO), there is formation of new subperiosteal cancellous bone at the
distal ends of long bones. Patients present with periarticular pain, polyarthralgia, and
painful swellings at the wrists, knees, elbows, and ankles. On examination, there is
clubbing of the fingers and toes. There may also be localized articular erythema,
tenderness, swelling, and even effusions. X-rays may reveal subperiosteal bone
formation with periosteal elevation seen as thickening and detachment of the
periosteum. Lambert-Eaton myasthenic syndrome is an immune-mediated disorder of
neuromuscular transmission resulting in impaired release of acetylcholine from nerve
terminals. Patients complain of weakness of the scapular and pelvic girdles muscles and
the resultant difficulties in performing activities such as climbing stairs. Other symptoms
such as dry mouth, paresthesias, and sexual impotence due to autonomic dysfunction
may also be present. On examination, there is proximal muscle weakness. Deep tendon
reflexes are decreased or absent. Horner's syndrome is due to a bronchogenic
carcinoma tumor mass extending to the sympathetic chain. Horner's syndrome
comprises of miosis, ptosis, enophthalmos, and hemianhydrosis. In addition, the patients
may also have atrophy of hand muscles. Cushing's syndrome may develop as a result of
the small cell carcinomas secreting ectopic adrenocorticotropic hormone (ACTH).
Patients may complain of bruising easily. On examination, they may have red-purplish
striae, especially over their abdomen, in addition to the typical moon facies and truncal
obesity. They may also have muscle wasting, especially of the proximal limb girdle
muscles. Laboratory investigations reveal hypokalemia and high plasma ACTH, as well
as increased serum and urine cortisol. References: 1. Harrison's Principles of Internal
Medicine 2. The Merck Manual 3. Sher T, Dy GK, Adjei AA. Small cell lung cancer. Mayo
Clin Proc. 2008 Mar;83(3):355-67. 4. Kantarjian H, Koller CA, Wolff RA, et al. The MD
Anderson Manual of Medical Oncology. McGraw-Hill Professional, 2006:1037

26.A 38-year-old man presents to the Emergency Department with the complaint of pain
and inability to extend his middle finger DIP joint following a sudden jamming type injury
when attempting to catch a football thrown by his son 3 hours ago. His radiographs are
normal. What is his diagnosis based on this history?
a) Swan neck deformity
b) Mallet finger
c) Boutonniere deformity
d) DIP dislocation
e) Trigger finger

Rationale: A mallet finger deformity is due to the rupture of the extensor tendon at the
base of the dorsal distal phalanx of any digit of the hand. Patients will report pain
dorsally at the DIP joint with the inability to actively extend the DIP joint. Swan neck
deformity may be the result of a mallet finger but only over time. Since this injury is
acute, swan neck deformity can't be the answer. Boutonnière deformity is a result of
rupture of the central portion of the extensor tendon at its insertion onto the middle
phalanx. It results in flexion of the PIP joint and hyperextension of the DIP joint. DIP
dislocation would be evident on radiographs. Trigger finger is the result of thickening of
the A1 pulley of the synovial sheath or flexor tendon itself at the entrance to the synovial
sheath. This causes the tendon to "catch" when the finger is flexed and then to "snap"
like pulling a "trigger" when trying to extend the finger. References: 1. Seiler III, JG.
Mallet Finger. In: Griffin LY, ed. Essentials of Musculoskeletal Care, 3rd ed. Rosemont,
IL: American Academy of Orthopaedic Surgeons; 2005:374. 2. Peterson L. Renström
P. Sports Injuries: Their Prevention and Treatment. Informa Health Care. 2001:201 3.
Tubiana R. Gilbert A. Tendon, Nerve and Other Disorders Taylor & Francis. 2005:36

27.A 37-year-old woman presents with a history of right-sided facial weakness and
periauricular discomfort since she awoke this morning. She is afebrile. What is the most
likely diagnosis?
a) Trigeminal neuralgia
b) Bell's Palsy
c) Multiple sclerosis
d) Myasthenia gravis
e) Primary lateral sclerosis

Rationale: The correct answer is Bell's palsy, which is a condition with typically sudden
onset that affects the facial nerve, causing unilateral facial weakness. Trigeminal
neuralgia presents with sharp pain on one side of the mouth that radiates to the
ipsilateral ear, eye, or nostril. Multiple sclerosis is a demyelinating disorder, causing a
multitude of symptoms that typically includes diplopia or blurred vision early on, then an
insidious onset of progressive weakness, numbness, and/or tingling in the extremities.
Myasthenia gravis commonly presents with ptosis and diplopia, as well as difficulty
swallowing, fatigue, and muscle weakness. Primary lateral sclerosis is an upper motor
neuron disease that causes limb weakness, stiffness, and fasciculations. References: 1.
McPhee SJ, Papadakis MA. Current Medical Diagnosis and Treatment. 47th ed. USA.
McGraw-Hill. 2008:881-896. 2. Tiemstra JD, Khatkhate N. Bell's palsy: diagnosis and
management. Am Fam Physician. 2007 Oct 1;76(7):997-1002. 3. Dufour SK. An unusual
case of stabbing eye pain: a case report and review of trigeminal neuralgia. Optometry.
2002 Oct;73(10):626-34.

28.A 34-year-old woman received a renal transplant 18 months ago for her diabetic
kidney disease. Her transplant is functioning well. Social and family histories are
negative for alcohol or substance abuse. Pre-transplant, her stress echocardiogram
showed no ischemic defects or arrhythmias and mild concentric left ventricular
hypertrophy.Attention to which of the following preventive measures will prevent one of
the leading causes of death in this patient?
a) Blood pressure and lipid control
b) Contraceptive counseling
c) Advise regular seat belt use
d) Increased consumption of grapefruit
e) Screening testicular ultrasounds

Rationale: Attention to blood pressure and lipid control will help prevent myocardial
infarctions, one of the leading causes of death in transplant patients. Her diabetes also
increases her risk of cardiac disease. Lipid abnormalities often occur as a complication
of immunosuppression (Brenner, chapter 65) and should be routinely evaluated in at risk
diabetic and transplant patients. The leading causes of death in transplant patients are
myocardial infarctions, cardiac arrest, septicemias, and cancers (USRDS). Family
planning and contraception may now become an issue in this patient after her transplant,
whereas pre-transplant her fertility was quite low. This preventive measure does not
address any of the leading causes of death in renal transplant recipients. Seat belt use is
meant to prevent trauma from motor vehicle accidents. Although this preventive
measure is suggested for all patients, it is not specific to mortality risk for renal transplant
patients. Grapefruit juice may alter levels of tacrolimus by altering its metabolism
(MICROMEDEX) and is therefore not suggested for this patient. Inconsistent intake
could translate into variable levels of immunosuppression and possibly rejection or
overmedication. Renal transplant patients are at increased risk for cancer, including ano-
gential and urological cancers, dermatological cancers and hematologic cancers (Lutz).
Although the US Preventive task force does not recommend screening for testicular
cancer in asymptomatic adolescent and adult males (AHQR website), testicular exams
may be prudent in male transplant recipients; this suggestion does not apply to female
patients. References: 1. MICROMEDEX Healthcare Series Copyright 1974-2009
Thomson Reuters All rights reserved 2. Lutz J. Heemann U. Tumours after kidney
transplantation. Current Opinion in Urology. 13(2):105-9, 2003 Mar. 3. Kapoor A.
Malignancy in kidney transplant recipients. Drugs. 68 Suppl 1:11-9, 2008. 4. Pedotti P.
Cardillo M. Rossini G. Arcuri V. Boschiero L. Caldara R. Cannella G. Dissegna D. Gotti
E. Marchini F. Maresca MC. Montagnino G. Montanaro D. Rigotti P. Sandrini S. Taioli E.
Scalamogna M. Incidence of cancer after kidney transplant: results from the North Italy
transplant program. Transplantation. 76(10):1448-51, 2003 Nov 27. 5.
http://www.ahrq.gov/clinic/uspstf/uspscolo.htm 6.
http://www.usrds.org/2008/ref/H_Mortality_&_Causes_of_Death_08.pdf 7. Brenner:
Brenner and Rector's The Kidney, 8th Edition, Copyright 2009 Elsevier, Chapter 65.

29.A 32-year-old woman presents to your clinic with the complaint of dominant right
hand pain and paresthesia of the thumb, index, and middle fingers for the past 3 months.
She works in a retail sales office where she spends much of the day on the computer
keyboard typing. What associated physical finding may indicate an advanced Carpal
Tunnel Syndrome?
a) Thenar atrophy
b) Hypothenar atrophy
c) Weakness on extension of the DIP joints
d) Decreased gap to 2 mm on 2-point discrimination test
e) Weak radial artery pulse

Rationale: Compression of the median nerve in Carpal Tunnel Syndrome (CTS) results
in paresthesias, pain, and sometimes paralysis in the median nerve distribution pattern
(palmar surface of the thumb, index, middle, and radial side of the ring fingers). When
the nerve is severely compressed for a long period of time it can also result in the motor
component of the nerve being compromised. The thenar muscles, if not supplied with
adequate nerve motor impulse, will begin to atrophy. Hypothenar atrophy would be an
indication of ulnar nerve compression seen in entrapment syndromes either at the elbow
or wrist. Weakness of digital extension is an indication of radial nerve problems, which is
more likely to be seen with humeral shaft fractures (although it is not very common). The
2-point discrimination test distance widens with CTS to over 15 mm of spread. The radial
artery lies outside of the carpal tunnel and therefore is unaffected in CTS. References: 1.
Seiler III, JG. Fractures of the Metacarpals and Phalanges. In: Griffin LY, ed. Essentials
of Musculoskeletal Care, 3rd ed. Rosemont, IL: American Academy of Orthopaedic
Surgeons; 2005:321. 2. Mallette P, Zhao M, Zurakowski D, Ring D. Muscle atrophy at
diagnosis of carpal and cubital tunnel syndrome. J Hand Surg [Am]. 2007 Jul-
Aug;32(6):855-8 3. Taylor G. The Board Buster Clinical Cases: Steps 2 and 3. Lippincott
Williams& Wilkins. 2004:33

30.A 25-year-old sexually active man notices a painless sore on his penis. 2 weeks later
he goes to his doctor. On questioning, he denies any discharge or pain with urination.
On physical examination, the sore is still present and he is afebrile. He is found to have
palpable inguinal lymph nodes. The lymph nodes are enlarged, but painless. Dark-Field
examination is done on an exudate from the lesion. The results of the Dark-Field
examination are positive. Question: What is the most likely diagnosis?
a) Syphilis
b) Condyloma acuminatum
c) Herpes genitalis
d) Chancroid
e) Gonorrhea

Rationale: This man has signs and symptoms of primary syphilis: a painless chancre
along with painless regional lymphadenopathy. Dark-Field examination of a lesion will
show the spirochetes in the majority of the cases. The ulcer seen with chancroid would
be painful. There would be painful tender nodes with chancroid. Gonorrhea can present
with symptoms of urethritis. A purulent discharge and dysuria are consistent with
gonorrhea. Intracellular diplococci on a Gram stain of the urethral discharge is also
consistent with the diagnosis of gonorrhea. The lesions caused by herpes genitalis are
vesicular lesions on an erythematous foundation. Soft painful adenopathy can be seen
with herpes genitalis as well as fever. Condyloma acuminatum is the cause of genital
warts. These are caused by human papilloma virus. Dark-Field examination is not done
on genital warts. References: 1. Andreoli, Tomas E., M.D., Carpenter, Charles, C.J.,
M.D., Griggs, Robert C., M.D., Benjamin, Ivor J., M.D. Andreoli and Carpenter’s
Cecil Essentials of Medicine. 7th edition. Philadelphia, PA. 2007. 2. Eccleston K, Collins
L, Higgins SP. Primary syphilis. Int J STD AIDS. 2008 Mar;19(3):145-51. 3. Peate I.
Syphilis: clinical presentation, diagnosis and treatment. Nurs Stand. 2007 Nov 14-
20;22(10):48-55; quiz 58.

31.A 23-year-old woman presents with a painful right arm. Tissue pressure in the arm is
55 mmHg. There is also swelling and tenderness to the touch. Patient has extreme pain
when the fingers of the right hand are flexed. There is no erythema present and no
purulent exudate is seen. What is the treatment for this patient's symptoms?
a) Antibiotics
b) Steroids
c) Surgery
d) Watchful waiting
e) Immobilization

Rationale: Compartment syndrome is compression of the blood vessels and nerves that
can occur in the extremities. As a result, nerve and muscle damage can result due to
decreased blood flow to the affected area. The muscles of the extremities are separated
into compartments by fascia. The fascia cannot expand and when pressure increases
inside of the compartment, the enclosed tissues can become compressed. The pressure
can increase to the extent that the blood flow becomes blocked. If the pressure is not
released, this can lead to permanent damage. Necrosis of the tissue can occur and will
require amputation of the limb. Compartment syndrome can result from trauma
(including long bone fractures), burns, surgery, hemorrhage from a large blood vessel, or
it can be the result of casts or bandages that have been placed too tightly. It is most
common in the forearms and lower legs; however, any part of an extremity may be
affected. A chronic form of compartment syndrome can be seen in runners. Symptoms
include pain, decreased sensation, pallor of the skin, and weakness. For example, the
patient may feel pain in the calf when the ankle is flexed. The pain may not be relieved
with pain medication or when the limb is elevated. This type of pain is a hallmark of
compartment syndrome. Diagnosis is usually made through examination. The patient will
experience pain when the affected muscle is moved. The skin over the affected area
may appear tight and shiny. There may also be swelling. Measuring the pressure in the
compartment will confirm the diagnosis. If the pressure within the compartment is greater
than 45 mmHg, the diagnosis is confirmed. Treatment is usually surgery. An incision is
made through the fascia in order to release the pressure. The incision is left open and
closed later. If the pressure is relieved quickly, the patient's prognosis is good. If the
pressure is not relieved, nerve damage can result in approximately 12-24 hours.
Permanent nerve damage is a complication that can result if the condition is left
untreated. References: 1. Tintinalli's Emergency Medicine: A Comprehensive Study
Guide, 6th Edition, Section 23, Chapter 278. 2. Current Surgical Diagnosis and
Treatment, 12th Edition, Chapter 42.

32.A 27-year-old man has several disfiguring nodules on his skin. He has about a dozen
café au lait spots ranging in size from 2 cm and upward on his trunk and armpits. In
addition, he has vertigo, weakness in his facial muscles, numbness in his face, and
atrophy of the muscles of mastication. This condition seems to run in his family, as his
brother and his father have a similar condition. What cranial nerve is responsible for the
changes in the muscles of mastication?
a) Cranial nerve V
b) Cranial nerve VII
c) Cranial nerve VIII
d) Cranial nerve IX
e) Cranial nerve X

Rationale: This patient has neurofibromatosis (Von Recklinghausen's multiple


neurofibromatosis). The multiple neurofibromas and café au lait spots that this man
has are consistent with neurofibromatosis. Café au lait spots are the pigmented skin
lesions seen with neurofibromatosis. Neurofibromatosis is inherited in an autosomal
dominant manner. Tumors of the peripheral nerves (either spinal nerves or cranial
nerves) can also be seen with neurofibromatosis. Cranial nerve V is the trigeminal nerve.
This nerve is responsible for sensory innervation to the face. It is also responsible for
motor innervation to the muscles of mastication. Cranial nerve VII is the facial nerve. It is
responsible for innervation of the muscles of facial expression. It carries sensation from
taste on the anterior 2/3rd of the tongue. It is also responsible for parasympathetic
innervation to the sublingual gland, the submandibular gland, mucus glands in the nasal
cavity, and the lacrimal gland. Cranial nerve VIII is the acoustic (or vestibulocochlear)
nerve. This is a sensory nerve responsible for hearing and vestibular sensations. Cranial
nerve IX is the glossopharyngeal nerve. The sensory fibers of this nerve are responsible
for taste from the posterior third of the tongue and the carotid sinus. It has a motor
component to some muscles involved with swallowing. It carries parasympathetic fibers
to the parotid gland. Cranial nerve X is the vagus. This has sensory, motor, and
parasympathetic functions. It is motor to muscles in the pharynx and larynx and is
involved in swallowing. It carries parasympathetic fibers to much of the viscera in the
thorax and abdomen. It carries sensory information from thoracic and abdominal organs.
References: 1. Andreoli, Tomas E., M.D., Carpenter, Charles, C.J., M.D., Griggs, Robert
C., M.D., Benjamin, Ivor J., M.D. Andreoli and Carpenter's Cecil Essentials of Medicine.
7th edition. Philadelphia, PA. 2007. 2. Moore, Keith L, Agur, Anne M.R. Essential Clinical
Anatomy. 1996. Baltimore, MD; Williams & Wiilkins.

33.Your patients, a 21-year-old female and a 25-year-old male, have come to you for
counseling a few weeks after their marriage and honeymoon. They are concerned about
dyspareunia and would like for you to explain its causes and how they might deal with it.
You explain that dyspareunia may be characterized by
a) inadequate lubrication
b) its equal occurrence in both sexual partners
c) vaginismus not being causative
d) amenorrhea being statistically causative
e) a complete lack of pain during intercourse

Rationale: ?Dyspareunia, or painful sexual intercourse, may be caused by one, or a


combination of, several factors. Vaginal dryness with inadequate lubrication, due to
inadequate sexual stimulation or urinary tract irritation may also be contributory. Women
with pelvic inflammatory disease, urinary tract infections, or other severe organic
disorders may present with dyspareunia as a relatively minor manifestation of underlying
disease. Urinary tract infections are commonly caused by Escherichia coli infection.
Additionally, other gram-negative rods such as Proteus, Klebsiella and Enterobacter,
account for a smaller proportion of uncomplicated infections. When lower urinary tract
infection is suspected, the physician must also rule out upper tract infection. Amenorrhea
is not statistically causative of painful sexual intercourse. A female patient may present
with complaints of painful intercourse when she is actually trying to talk to her doctor
about marital discord or sexual problems of a psychological nature. Normal reproductive
and sexual function are complex issues that are a result of the orchestration of
integrated actions of the endocrine system, the central nervous system and the
reproductive organs. In many cases, the physician must ascertain the underlying cause
of presenting sexual difficulties. Vaginismus may, in fact, be causative. The invasion of
the uterosacral ligaments by abnormal deposition of endometrial tissues on the
uterosacral ligaments causes referred pain in the sacral region and would not present
with the symptomology described.

34.A 12-year-old girl with recently diagnosed bronchial asthma is prescribed 1-2 puffs of
salbutamol metered dose inhaler (MDI). Which of the following is the primary mechanism
of action (in asthma) of the drug prescribed to this patient?
a) Antimuscarinic action and decrease of vagal tone in the airways
b) Stimulation of beta2 adrenergic receptors on the bronchial smooth muscle
c) Adenosine antagonism and non-selective inhibition of phosphodiesterase
d) Activation of glucocorticoid receptor to produce anti-inflammatory effect
e) Stabilizing effect on the sensitized mast cell degranulation

Rationale: ? Salbutamol is an agonist of beta2 adrenergic receptors and acts by


stimulating beta2 adrenergic receptors on the bronchial smooth muscle. Activation of the
beta2 adrenergic receptors causes relaxation of the bronchial smooth muscle due to the
generation of intracellular cAMP. In addition to bronchodilation, they also cause
mucociliary clearance and inhibit release of mediators from mast cells. They are the
mainstay therapeutics in the treatment of acute bronchial asthma. They are administered
through inhalation using MDI or nebulizer and can be given orally. The adverse effects
are nervousness, fine tremors, and tachycardia. Anticholinergics such as ipratropium act
by inhibiting acetylcholine action on the muscarinic receptor in the bronchial airways.
Ipratropium, a muscarinic antagonist, causes bronchodilation by the inhibition of vagal-
mediated bronchoconstriction. Steroids are useful in bronchial asthma due to their anti-
inflammatory effect through the action on the glucocorticoid receptor. They inhibit
various inflammatory-mediator cells and decrease the production of cytokines and other
inflammatory mediators involved in bronchial asthma. The exact mechanism of action of
methyl xanthines such as theophyllines is unclear, although their use in bronchial
asthma is attributable to its adenosine antagonistic effects and non-selective inhibition of
phosphodiesterase. By preventing histamine release, mast cell membrane stabilizing
agents such as chromoglycate attenuate bronchospasm. These drugs primarily act by
inhibiting sensitized mast cells degranulation following exposure to antigens.
References: 1. Respiratory Disease. In: Reid JL, Rubin PC, Walters MR. Lecture Notes:
Clinical Pharmacology & Therapeutics. Blackwell Publishing; 2006:111-112. 2. Asthma,
hay fever and anaphylaxis. In: Neal MG. Medical Pharmacology at a Glance, 5th ed.
Blackwell Publishing; 2005: 29. 3. Philips RS, Ball CM. Evidence based on-call
Emergency Medicine. Elsevier. 2004; 106 4. Tofield C, Milson A, Chatu S. Hands-on
guide to Pharmacology. Wiley Blackwell. 2004; 46.

35.A 45-year-old man presents to the emergency room complaining of a 3-month history
of weakness of his shoulder muscles. He also reports tingling sensations in his feet. He
is not taking any medications. He is a divorced waiter who lives alone and drinks alcohol
everyday. On examination his BP is 140/90mm/Hg and PR 92/min. He is obese and
pale, with a tinge of jaundice. He has telangiectasia. Laboratory investigations reveal
elevated GGT levels. An electromyogram is normal. What is the most likely cause of his
muscle weakness?
a) Alcoholism
b) Hyperthyroidism
c) Cushing's disease
d) Hypothyroidism
e) Systemic Lupus Erythematosus

Rationale: Muscle weakness due to alcoholism affects the proximal muscles, though the
distal muscles may also be affected. Patients complain of difficulties doing everyday
activities, such as combing their hair. They may also complain of tingling sensations due
to peripheral neuropathy. On examination, they may have telangiectasia and other signs
of chronic alcohol abuse. Laboratory investigations may reveal low hemoglobin and
vitamin B12 levels and elevated transaminase and gamma GT (GGT) levels. The
electromyogram is normal. A muscle biopsy shows nonspecific myopathic changes such
as atrophy and regeneration of muscle fibres. Muscle weakness due to hyperthyroidism
affects the proximal and bulbar muscles. Patients are usually in their forties. They have
other symptoms of hyperthyroidism, such as excessive perspiration, emotional lability,
and weight loss. On examination, they may have tachycardia, exophthalmos, and fine
tremors. Laboratory results reveal elevated T3 and T4 levels. Electromyogram reveals
myopathic motor unit action potentials. Muscle biopsies are usually normal. Muscle
weakness due to hypothyroidism affects the proximal muscle groups. Patients complain
of difficulties doing everyday activities, such as combing their hair. It usually affects
patients in the 30 to 49 year age group. They may also have other symptoms of
hypothyroidism, such as menorrhagia and weight gain. On examination, they may have
a goiter and bradycardia. Laboratory results reveal elevated thyroid stimulating hormone
(TSH) and Creatine kinase. Muscle biopsies reveal glycogen accumulation and
nonspecifice myopathic changes, such as atrophy and regeneration of muscle fibers.
Muscle weakness due to Cushing's disease affects the proximal muscle groups. Patients
complain of difficulties in everyday activities, such as rising from a chair. They may have
other symptoms associated with glucocorticoid excess, such as easy bruising and
emotional lability. On examination, they may have truncal obesity, the characteristic
buffalo hump, and striae. Laboratory results reveal a normal creatine kinase and
elevated free cortisol in the urine. Electromyogram shows myopathic motor unit action
potentials. Muscle biopsies show selective atrophy of type II muscle fibers. Muscle
weakness due to systemic lupus erythematosus (SLE) affects the proximal muscle
groups of the shoulder and pelvic girdles. Patients have difficulties doing everyday
activities, such as walking. In addition to the weakness, they may also present with other
signs and symptoms of SLE, such as a malar rash and joint pains. Laboratory results
reveasl positive anti-nuclear antibodies (ANA) and depressed C3 complement levels.
Muscle biopsies reveal atrophy of type II muscle fibers. References: 1. The Merck
Manual 2. Harrison's Principles of Internal Medicine 36.A 24-year-old college graduate
presents with paroxysmal attacks of headaches, anxiety, and excessive sweating for the
past month. On examination, there is no pallor, icterus, edema, or lymphadenopathy. His
BP is 150/78 mmHg. Biochemical investigations reveal an elevated blood sugar level
and increased levels of urinary VMA (vanillyl mandelic acid). He was diagnosed with
pheochromocytoma, a diagnosis that is confirmed by a CT scan showing a mass on the
left. On being informed of the condition and the treatment options available to him, he
consents to surgery. During surgery, after location of the adrenal tumor, the vein that
needs to be ligated before excision is the left adrenal vein. The left adrenal vein drains
into which of the following veins?
a) Inferior vena cava
b) Renal vein
c) Gonadal vein
d) Azygous vein
e) Inferior mesenteric vein
Rationale: The venous drainage of the adrenal gland is through a single vein. The left
adrenal vein drains into the left renal vein, whereas the right adrenal vein drains into the
inferior vena directly. The patient here has an adrenal mass on the left. The
pheochromocytoma (PCC) tumors arise from chromaffin cells of the adrenal medulla.
The presentations of PCC vary with the production of active metabolites. The episodic
alpha-adrenergic hypersecretion results in the manifestations of intermittent malignant
hypertension. PCC commonly develops in young-to-middle-aged adults. The classic
triad is episodic headache, tachycardia, and diaphoresis. Hypertension is the most
common clinical sign of pheochromocytoma. The adrenal gland is supplied by 3 arteries:
the superior suprarenal arteries (a branch of inferior phrenic artery), middle suprarenal
arteries (directly from the aorta), and inferior suprarenal arteries (from renal arteries).
The venous drainage of the adrenal gland must be ligated before handling the tumor to
prevent the entry of enormous amounts of adrenaline into the blood stream. Failure to
ensure proper ligation can lead to life threatening complications. References: 1. Soon,
Patsy S.H.; Yeh, Michael W.; Sywak, Mark S.; Sidhu, Stan B.; Use of the Ligasure
Vessel Sealing System in Laparoscopic Adrenalectomy: Surgical Technique; ANZ
Journal of Surgery. 76(9):850-852, September 2006. 2. Saif Al-Sobhi, MD, Reinhard
Peschel, MD, Christine Zihak, MD, Georg Bartsch, MD, Hartmut Neumann, MD,
Günter Janetschek, MD; Journal of Endourology; Laparoscopic Partial Adrenalectomy
for Recurrent Pheochromocytoma after Open Partial Adrenalectomy in Von Hippel-
Lindau Disease; Apr 2002, Vol. 16, No. 3: 171-174 3. Clinically Oriented Anatomy by
Keith L. Moore;Third edition Williams and Wilkins, 1992.

37.Your last appointment of the day is with a couple in their nineties. The husband wants
to consult with you about his wife and how long she has to live. You have been treating
his wife for the past 3 years for general decline in health. His wife does not have any
specific health problems. What will you tell him?
a) Elicit the reason for his concern
b) You shouldn’t think about your wife dying.
c) Have you made funeral arrangements?
d) Are you hoping your wife will not live much longer?
e) I don’t know how long she will live.

Rationale: Opening a dialogue with him about his concerns for his wife will lead the
conversation in the direction in which you may reassure him without telling him how long
his wife will live. "You shouldn't think about your wife dying" shuts down the
conversation. "Have you made funeral arrangements?" is not appropriate at this time.
"Are you hoping your wife will not live much longer?" as before, asking this question
shuts down the conversation. Stating that you do not know how long she will live may
stop him from asking more questions that concern him. Reference: 1. Primary Care
Medicine: Office Evaluation and Management of the Adult Patient; Allan H. Goroll,
Lawrence A. May, Albert G. Mulley, Jr.

38.You are auscultating a 32 year-old African American male while simultaneously


measuring his ECG. The first heartsound corresponds with what activity on an ECG?
a) The beginning of the P wave
b) The P-Q interval
c) The QR interval
d) The RS interval
e) The end of the T wave
Rationale: The first heartsound, the turbulence associated with the closure of the A-V
valves, occurs at the declination of the R wave and terminates approximately at the end
of the S wave.

39.A 66-year-old man presents with a 2-month history of numbness of his lower limbs
and a feeling of being tired. He has been a strict vegetarian for the past 2 years. On
examination he is pale with an ataxic gait and diminished proprioceptive sensation in his
legs. A peripheral blood smear reveals macrocytic red cells. What is the most likely
diagnosis?
a) Iodine deficiency
b) Zinc deficiency
c) Copper deficiency
d) Folic acid deficiency
e) Cobalamin deficiency

Rationale: Patients with vitamin B12 or cobalamin deficiency can present with
symptoms and signs of anemia such as fatigue, light-headedness, syncope, and pallor,
as well as neurological ones such as ataxia, paraesthesias, and diminished
proprioceptive and vibratory sensations in the lower limbs. Laboratory investigations
reveal macrocytic red blood cells. Causes include pernicious anemia, blind loop
syndrome, fish tapeworm infestation, and vegetarian diets. Dietary sources of vitamin
B12 include liver, beef, eggs, and milk. In zinc deficiency patients can present with
hypogeusia or decreased taste sensation, anorexia, delayed sexual maturation, night
blindness, and hair loss. On examination they have alopecia, growth retardation,
delayed sexual maturation, and hypogonadism. Laboratory investigations may reveal
hypospermia. Causes include malabsorption states, alcoholism, and prolonged
parenteral nutrition. Dietary sources of zinc include beef, liver, eggs, and oysters. In
iodine deficiency, patients can present with an anterior neck swelling that rises with
deglutition. This is usually a colloid goiter, as the thyroid hypertrophies as it tries to
concentrate iodide in itself. Though most of these patients are euthyroid, some may
develop hypothyroidism. Cretinism can develop in infants as a result of iodine deficiency
as well as impaired brain development and fetal growth. Causes include inadequate
dietary intake. Dietary sources of iodine include iodized table salt, seafood, eggs, and
dairy products. Menkes' syndrome is an inherited copper deficiency caused by mutations
in an X-linked gene. It occurs in male infants and is characterized by mental retardation,
kinky hair, hypopigmentation, and vascular aneurysms. Laboratory investigations reveal
hypocupremia and decreased circulating ceruloplasmin. Other causes of copper
deficiency include severe malabsorption, infants with persistent diarrhea fed on milk
diets, copper-free total parenteral nutrition, and excess intake of a zinc salt dietary
supplements, which can interfere with copper absorption. Dietary sources of copper
include organ meats, oysters, nuts, dried legumes, and whole grain cereals. Patients
with folic acid deficiency present with symptoms of anemia such as fatigue, weakness,
and syncope. On examination they are pale. Laboratory investigations reveal macrocytic
red blood cells and hemoglobin less than 12g/dl. Causes include malabsorption and
inadequate intake by chronic alcoholics and malnourished individuals, as well as
increased demand in pregnancy and chronic hemolytic anemias. Dietary sources of folic
acid include green leafy vegetables, liver, and yeast. References: 1. The Merck Manual
2. Harrison's Principles of Internal Medicine

40.A 40-year-old type 2 diabetic patient arrives with her fasting blood sugar is 200
mg/dL. The patient informs that she has not been adhering to a strict diabetic diet
pattern. She is redirected to a dietician who gives her a new diabetic diet chart. The
patient is advised to cut down on her high carbohydrate diet. The aerobic conversion of
1 mole of glucose to 2 moles of pyruvate is accompanied by the generation of how many
moles of adenosine triphosphate (ATP)?
a) 2
b) 4
c) 8
d) 10
e) 12

Rationale: The aerobic conversion of 1 mole of glucose to 2 moles of pyruvate is


accompanied by the generation of 4 moles of ATP and 2 moles of nicotinamide adenine
dinucleotide phosphate (NADH). This process of aerobic glucose glycolysis requires 2
equivalents of ATP to activate it and results in the production of 4 equivalents of ATP
and 2 equivalents of NADH. The reaction is as follows: Glucose + 2ATP +2 NAD+ → 2
Pyruvate + 4ATP + 2 NADH Hexokinase is the first enzyme in the aerobic glycolysis
pathway. It catalyzes the phosphorylation of glucose, using ATP, to form glucose-6-
phosphate. The subsequent ATP utilizing reaction is the phosphorylation of fructose-6-
phosphate by phosphofructokinase to fructose 1, 6-bisphosphate. Thus, in the first
phase, glucose is converted to fructose 1, 6-bisphosphate using 2 equivalents of ATP. In
the second phase, fructose 1, 6-bisphosphate is converted to pyruvate with the
subsequent generation of 4 equivalents of ATP and 2 equivalents of NADH. Reference:
1. Stavrianeas S, Silverstein T. Teaching glycolysis regulation to undergraduates using
an electrical power generation analogy. Adv Physiol Educ. 2005 Jun; 29(2):128-30.

41.While reviewing a study regarding a new medication called Vasolaxin, you read the
following statement claimed by researchers of the study: "In clinical trials, Vasolaxin
lowered diastolic blood pressure in patients with chronic hypertension by an average of
15% (p-value of <0.10)." Which of the following statements is true regarding this claim?
a) Less than 10% of patients experienced no effect from Vasolaxin.
b) Greater than 90% had favorable clinical results from Vasolaxin.
c) The observed value has a less than 10% chance of occurring by chance
alone.
d) There is a 90% chance of reproducibility of this experimental study.
e) Less than 10% of subjects fell outside of 2 standard deviations of the mean for
diastolic blood pressure.

Rationale: The p-value of a study is the probability of obtaining a result at least as


extreme as the one that was actually observed, assuming that the null hypothesis is true.
Or stated more simply, the p-value is the expression of the chance that an observed
outcome is the product of random chance alone. Thus, a p-value of < 0.10 means that
the observed outcome has a < 10% chance that it was a random occurrence. Thus, the
lower the p-value, the more statistically significant the result. References: 1. Michael J.
Campbell, David Machin, and Stephen J. Walters. Medical Statistics: A Textbook for the
Health Sciences. Aug 27, 2007. 2. Wolfgang Dahnert. Radiology Review Manual. 6th
Edition. April 2007. 3. David Machin and Michael J. Campbell. The Design of Studies for
Medical Research. May 6, 2005

42.In a classical conditioning experiment, an animal is taught to fear a red light by


pairing red light with the presentation of a mild shock. The experimenter then finds that
the animal shows a fear response to a green light as well. This is called:
a) Extinction
b) Spontaneous recovery
c) Higher-order conditioning
d) Stimulus discrimination
e) Stimulus generalization

Rationale: ? Learning is defined as a relatively permanent change in behavior as a


result of practice. Ivan Pavlov was an early 20th century Russian physiologist who
conducted an experiment on dogs to demonstrate classical conditioning, also known as
association learning. He noted that the reflex response to food, salivation, would be
elicited by dogs even before the actual presentation of the food by some event that
signaled food such a sound. To investigate the learning of the association between a
neutral stimulus and food, Pavlov set up the classic experiment in which he would ring a
bell immediately preceding the time that the food appeared before a dog, and thereby
condition the dog to salivate to the sound of the bell. The dog was fitted with an
apparatus that would collect and measure salivation. Lastly, the bell would be presented
alone, without food, and salivation would be measured. The procedure can be better
understood when it is divided into three phases as follows: Phase 1: Before
Conditioning. This is to demonstrate that the bell is indeed a neutral stimulus to start with
Ringing of Bell (Neutral Stimulus) elicits no response Food (Unconditioned Stimulus)
elicits salivation (Unconditioned Response; termed so because it is a reflex response
and is not acquired through learning) Phase 2: Acquisition. This is the phase in which
the bell and food are paired and the dog learns the association between the bell and
food Bell (Conditioned Stimulus) elicits salivation (as a result of being paired with the
unconditioned stimulus, i.e. food) Food (Unconditioned Stimulus) elicits salivation
(Unconditioned Response) Phase 3: Test for Conditioning. This is the phase in which the
bell is presented alone to demonstrate that learning has occurred Bell (Conditioned
Stimulus) elicits salivation (Conditioned Reflex, or Conditioned Response) The pairing of
the 2 stimuli in Phase 2 is the reinforcement for the conditioned response.
Reinforcement, by definition, is any event that increases the probability of a response. In
classical conditioning, the reinforcement is always the pairing of 2 stimuli. In this
experiment, if the bell and the food were no longer paired (that is, if the reinforcement
were removed), then the conditioned response would be extinguished. This is known as
extinction. However if the experimenter were to once again present the 2 stimuli
together, learning of the conditioned response would take a much shorter time. This is
known as spontaneous recovery. Stimuli that are very similar such as 2 bells with
different pitches, 2 lights of different wattage, or 2 different colored lights can sometimes
elicit similar conditioned responses. For example, a dog conditioned to bark at the sound
of a door bell may also bark at the sound of a telephone ringing. This is known as
stimulus generalization. Stimulus generalization involves associating a similar stimulus
with the original conditioned stimulus. Thus, there is a tendency for a conditioned
response to occur in the presence of a stimulus that is similar to the conditioned
stimulus. It is possible to learn to discriminate between 2 similar stimuli by presenting 1
of the stimuli paired with an unconditioned stimulus and the other alone, unpaired. This
is known as stimulus discrimination. Hence, in stimulus discrimination there is a
tendency for a response to be elicited by one stimulus and not another. Higher-order
conditioning refers to the use of a conditioned stimulus such as a bell in the place of an
unconditioned stimulus, and to pair it with a second neutral stimulus, such as a light. The
3 phases would be followed thus: The light elicits no response in Phase 1, while the bell
elicits a conditioned response. In Phase 2, the light and the bell are presented together
so that the association may be learned. Finally, in Phase 3, the light is presented alone
to demonstrate the conditioned response. References: 1. Rhea R. Kimpo, Edward S.
Boyden, Akira Katoh, Michael C. Ke and Jennifer L. Raymond; Distinct Patterns of
Stimulus Generalization of Increases and Decreases in VOR Gain; J Neurophysiol 94:
3092-3100, 2005. 2. Gewirtz JC and Davis M. Application of Pavlovian higher-order
conditioning to the analysis of the neural substrates of fear conditioning;
Neuropharmacology . 1998; 37(4-5):453-9.

43.A 16-year-old Hispanic girl presents to the hospital complaining of a 3-day history of
painful swellings in her groin. On examination, you note multiple yellow pustules around
the hair follicles. What is the causative organism?
a) Candida albicans
b) Trichomonas vaginalis
c) Haemophilus ducreyi
d) Varicella-zoster virus
e) Staphylococcus aureus

Rationale: The clinical presentation is suggestive of folliculitis, which is commonly


caused by the Staphylocccus aureus. Patients present with painful pustules around the
hair follicles. Treatment is with antibiotics such as dicloxacillin. Patients with Candida
albicans vaginal infection present with white, non-foul smelling discharge. On
examination, the vulva and vagina are erythematous with fissures and even lacerations,
as it is pruritic. Oral fluconazole or clotrimazole creams are used to treat it. Patient with
bacterial vaginosis, which is caused by Gardnerella vaginalis, present with a
malodorous, grayish vaginal discharge. On examination, the vulva and vagina are not
inflammed. Treatment is with metronidazole. Patients with chancroid, which is caused by
Haemophilus ducreyi, present with painful genital ulcers and suppurative inguinal
lymphadenopathy. Azithromycin is used for treatment. Patients with Herpes Zoster,
which is caused by the Varicella-zoster virus, present with painful vesicles in a unilateral
dermatomal distribution. Famciclovir is used for treatment. References: 1. The Merck
Manual 2. Harrison's Principles of Internal Medicine 3. www.cdc.gov

44.A 52-year-old woman is found to have an area of calcification on chest X-ray. It is


determined that she has dystrophic calcification, rather than metastatic calcification.
Which of the following conditions is associated with her condition?
a) Hyperparathyroidism
b) Immobilization
c) Coagulative necrosis
d) Milk-alkali syndrome
e) Sarcoidosis

Rationale: Dystrophic calcification is seen when the serum calcium level is within
normal limits. Dystrophic calcification is not the result from a defect with the handling of
calcium. Coagulative necrosis is associated with the occurrence of dystrophic
calcification. Metastatic calcification is seen when there is an elevated serum calcium,
secondary to some sort of abnormality in the handling of calcium. Hyperparathyroidism,
immobilization, milk-alkali syndrome, and sarcoidosis are among the conditions that can
result in an elevated calcium level and metastatic calcification. References: 1. Ghada El-
Hajj Fuleihan, MD., Nelly Rubeiz, MD. Dermatologic manifestations of parathyroid-
related disorders. Clinics in Dermatology (2006) 24, 281-288. 2. Raphael Rubin,
Emanuel Rubin Bruce Fenderson. Lippincott's Review of Pathology: Illustrated
Interactive Q&A, Lippincott Williams & Wilkins. 2006
45.An 18-year-old African American man presents to the hospital complaining of
bilateral pretibial pain for 1 day, which began while he was training for a marathon,
though he did not sustain any injury to his legs. He gives a history of similar body pains
since his childhood. He also reports that his older brother died of a similar illness at the
age of 22 years. On examination, he has mild pallor and a tower-shaped skull. On
successful management of his acute symptoms, what vaccine would you recommend for
him to receive?
a) Hepatitis A vaccine
b) Rabies vaccine
c) Pneumococcal vaccine
d) Lyme disease vaccination
e) Measles vaccination

Rationale: The clinical picture is suggestive of a vaso-occlusive painful crisis in a patient


with sickle cell disease. In adults, autosplenectomy usually occurs as a result of the
repeated infarctions and resultant fibrosis. This increases their susceptibility to
infections, especially from pneumococci. As a result, patients with sickle cell disease are
recommended to have the polyvalent pneumococcal vaccine. Other vaccinations
recommended for patients with sickle cell disease include: 1. Hepatitis B vaccine if they
are hepatitis B surface antigen negative 2. Yearly influenza vaccination Hepatitis A
vaccine is recommended for travelers to endemic areas such as tourists, missionaries,
and military personnel. Lyme disease vaccination is recommended for persons who live,
work, or recreate in tick-infested habitats. Measles vaccination is recommended for
persons attending college, health care workers, and travelers to foreign countries.
Rabies vaccine is recommended for high-risk groups such as laboratory workers,
veterinarians, and animal handlers. References: 1. The Merck Manual 2. Harrison's
Principles of Internal Medicine 3. Adamkiewicz TV, Silk BJ, Howgate J, at al.
Effectiveness of the 7-valent pneumococcal conjugate vaccine in children with sickle cell
disease in the first decade of life.Pediatrics. 2008 Mar;121(3):562-9 4. William BM,
Corazza GR. Hyposplenism: a comprehensive review. Part I: basic concepts and
causes. Hematology. 2007 Feb;12(1):1-13 5. Halasa NB, Shankar SM, Talbot TR, et al.
Incidence of invasive pneumococcal disease among individuals with sickle cell disease
before and after the introduction of the pneumococcal conjugate vaccine. Clin Infect Dis.
2007 Jun 1;44(11):1428-33. Epub 2007 Apr 18

46.An 18-month-old girl presents to the emergency room with a painful left arm. After
eliciting her history and performing an examination, it is determined that she has
suffered a pulled elbow with subluxation of the head of the radius. There is no evidence
of any other trauma on examination. The next step in treating this patient should be
a) Orthopedic consult to schedule open reduction
b) Discharge with oral analgesics and orthopedic follow-up in 1 week
c) Closed reduction in the emergency room
d) Admit the patient for observation
e) Call Child Protective Services as you suspect child abuse

Rationale: Nursemaid's elbow (annular ligament displacement or radial head


subluxation) is usually caused by an adult pulling or yanking on the outstretched arm of
a toddler. The radial head slips through the annular ligament, which is then displaced
towards the elbow joint. The child will usually hold the elbow partially flexed and will
refuse to move the arm. Reduction can readily be carried out by supination and
extension of the forearm followed by full flexion of the forearm. This restores the annular
ligament back to its normal anatomical position. Immobilization is not necessary and
usually the child regains normal use of the joint within 20 minutes. In this case, there is
no indication for open reduction. Discharging the patient with analgesics without
correcting the displacement will result in continued pain and lack of movement of the
joint. Observation will only delay the reduction. At this time, there is no indication for a
child abuse investigation as this is a relatively common injury and there are no other
signs of abuse. References: 1. Kaplan RE, Lillis KA. Recurrent nursemaid's elbow
(annular ligament displacement) treatment via telephone. Pediatrics. 2002 Jul;110(1 Pt
1):171-4. 2. Kunkler CE.Did you check your nursemaid's elbow? Orthop Nurs. 2000 Jul-
Aug;19(4):49-52; quiz 53-5

47.You are seeing a couple for the first time. They have come in for prenatal counseling.
They want to know if they should consider a home birth with a midwife. What is one of
the first things you would tell them about this option?
a) Home birth is a consideration; let’s discuss the pros and cons.
b) No, a home birth will end in tragedy.
c) The baby will be damaged if the baby is not born in a hospital.
d) Midwives do not know what they are doing.
e) Home birth is the only way to go.

Rationale: When counseling couples on where their baby should be born, it is important
to engage them in active conversation. Encourage the couple to explore all options with
pros and cons of each options explained. Telling them home birth will end in tragedy will
shut down any conversation. Telling them the baby will be damaged or that midwives
don't know what they're doing are opinions, not necessarily true statements. Expressing
that home birth is the only good option will end the conversation and not encourage the
clients to express their questions. Reference: 1. Primary Care Medicine: Office
Evaluation and Management of the Adult Patient; Allan H. Goroll, Lawrence A. May,
Albert G. Mulley, Jr.

48.A 78-year-old female who is a known case of congestive cardiac failure presents to
the clinic with complaints of ulcers on her right foot for the past 2 days. Examination
reveals bilateral pitting pedal edema with 2 small 2X2 cm venous ulcers at the medial
malleolus of her right foot along with increased jugular venous pressure (JVP),
hepatomegaly, and S3 gallop on cardiac auscultation. The patient is advised
compression stocking for the venous ulcer. Application of compression stocking helps
reduction of edema through which of the following hemodynamic mechanism?
a) Increasing capillary hydrostatic pressure
b) Increasing capillary permeability
c) Increasing tissue hydrostatic pressure
d) Increasing tissue oncotic pressure
e) Increasing venous pressure

Rationale: Application of external compression devices, such as pneumatic


compression garments and elastic wraps, helps in the reduction of edema by increasing
tissue hydrostatic pressure and enhancing venous return when applied over the area of
effusion and edema. Compression devices reduce venous reflux and promote venous
return, which is a significant factor in achieving ulcer healing. Increased capillary
hydrostatic pressure and capillary permeability promotes development of edema by
pushing fluid out into the tissue by the former and both fluid and solute by the latter.
Increasing venous pressure leads to the development of edema due to venous stasis.
Increasing tissue oncotic pressure also promotes edema by pulling fluid from the
capillary into the tissue bed. References: 1. Partsch H. Understanding the
pathophysiological effects of compression. In: Understanding compression therapy.
EWMA Position document. London: MEP Ltd, 2003: 2-4. 2. Tsang KKW, Hertel J, and
Denegar CR. Volume Decreases After Elevation and Intermittent Compression of
Postacute Ankle Sprains Are Negated by Gravity-Dependent Positioning. J Athl Train.
2003; 38(4): 320-324. 3. Yu GV, Schubert EK, Khoury WE. The Jones compression
bandage. Review and clinical applications. J Am Podiatr Med Assoc. 2002; 92(4):221-
31.

49.A 13-year-old girl is brought to her family doctor because her mother is concerned
about her weight loss, excessive exercise, surreptitious laxative use, and odd eating
habits. The doctor is alarmed: it is immediately apparent that she is abnormally thin and
weak. Her laboratory results are as follows: TEST RESULTS REFERENCE RANGE
calcium 9.8 mg/dL 8.4-10.2 mg/dL potassium 2.9 mEq/L 3.5-5.0 mEq/L sodium 139
mEq/L 135-145 mEq/L glucose (fasting) 65 mg/dL 70-110 mg/dL An EKG is performed.
What EKG finding would be most consistent with these laboratory results?
a) Osborne waves
b) Q waves
c) Peaked T waves
d) Sine wave pattern
e) U waves

Rationale: This girl has hypokalemia secondary to her laxative abuse. U waves can be
seen with hypokalemia. Osborne waves are seen with hypothermia. Q waves are seen
with myocardial infarction. Peaked T waves are seen with hyperkalemia. In severe
hyperkalemia, the EKG assumes a sine wave pattern. Reference: 1. Andreoli, Tomas E.,
M.D., Carpenter, Charles, C.J., M.D., Griggs, Robert C., M.D., Benjamin, Ivor J., M.D.
Andreoli and Carpenter's Cecil Essentials of Medicine. 7th edition. Philadelphia, PA.
2007.

50.A 22-year-old woman sees you for a physical examination. When questioning her for
the medical history, you discover that she has a history of rheumatic fever. Upon
listening to her heart, you detect a murmur. Which valvular abnormality did you probably
hear?
a) Mitral regurgitation
b) Aortic regurgitation
c) Mitral stenosis
d) Aortic stenosis
e) Tricuspid stenosis

Rationale: Mitral stenosis is the valvular abnormality that is seen most frequently as a
consequence of rheumatic fever. However, mitral regurgitation, aortic stenosis, aortic
regurgitation, and tricuspid stenosis can all also be seen as a consequence of rheumatic
fever. The mitral valve is the most affected valve as a consequence of rheumatic fever,
followed by the aortic valve, the tricuspid valve, and then the pulmonic valve. The
pulmonic valve is not involved very often. Reference: 1. Andreoli, Tomas E., M.D.,
Carpenter, Charles, C.J., M.D., Griggs, Robert C., M.D., Benjamin, Ivor J., M.D. Andreoli
and Carpenter's Cecil Essentials of Medicine. 7th edition. Philadelphia, PA. 2007.

You might also like